Fonctions de la variable réelle

image
Limite d’une fonction en un points puis théorèmes généraux sur les fonctions continues et outils de comparaison

On dit qu’une grandeur est la limite d’une autre grandeur, quand la seconde peut s’approcher de la première plus près que d’une grandeur donnée, si petite qu’on puisse la supposer...

D’Alembert.

Pour bien aborder ce chapitre

Nous poursuivons dans ce chapitre le travail commencé dans le précédent et nous allons étendre la notion de limite aux fonctions d’une variable réelle à valeurs réelles.

Si Leibniz parle de fonction (dans le sens de quantité dépendant d’une autre), Johann Bernouilli est le premier à parler de fonction d’une ordonnée , ce que Leibniz reprendra et les deux discuteront de la façon de désigner une fonction par un symbole.

En \(1718\), Bernouilli publie la première définition formelle. Euler la reprend en \(1748\) dans son Introductio in analysin infinitorum . Une fonction d’une quantité variable est une expression analytique composée de n’importe quelle façon de la quantité variable et de nombres ou quantités constantes. Mais dans les Institutiones calculi differentialis , il définit une fonction comme une variable dépendant d’une autre variable. Cette double vision va se poursuivre tout le \(18^{\textrm{ e}}\) siècle. On la retrouve, par exemple, chez Lagrange.

En \(1821\), dans son Cours d’analyse , Cauchy définit exclusivement une fonction comme une variable dépendant d’une autre variable. Mais en \(1822\), Fourier dans sa Théorie analytique de la chaleur , écrit En générale, la fonction f(x) représente une suite de valeurs ou ordonnées dont chacune est arbitraire [...]. On ne suppose pas que ces ordonnées soient assujetties à une loi commune. Elles se succèdent d’une manière quelconque, et chacune d’elles est donnée comme le serait une seule quantité. Pour autant, dans une démonstration sur la convergence d’une série, il suppose implicitement que toute fonction est continue.

Il faut attendre Lejeune-Dirichlet en \(1837\) pour avoir une définition moderne de la notion de fonction. Elle sera reprise presque mot pour mot par Riemann en \(1851\). Le point de vue d’Euler et Lagrange n’a cependant pas immédiatement disparu et Weierstrass considérait la définition de Dirichlet comme intenable et improductive. On peut voir le travail de Weierstrass comme une recherche pour obtenir la définition la plus utile de la notion de fonction.

Dans ce chapitre, après avoir introduit les notions de limite et de continuité en un point, nous réaliserons l’étude des propriétés locales des fonctions, c’est-à-dire des propriétés vraies dans un voisinage suffisamment petit d’un point donné. Dans la seconde partie du chapitre, nous énoncerons des théorèmes globaux, c’est-à-dire vrais sur un intervalle tout entier. Parmi ces théorèmes, quatre sont fondamentaux :

  1. L’image d’un intervalle par une fonction continue est encore un intervalle (ce théorème est équivalent au théorème des valeurs intermédiaires).

  2. L’image d’un segment par une application continue est un segment. (Ce théorème est équivalent au théorème du maximum, toute fonction continue sur un segment est bornée et atteint ses bornes).

  3. Le théorème de Heine qui aura des conséquences importantes dans la suite du cours.

  4. Le théorème de continuité de la bijection réciproque.

Vocabulaire

Dans tout ce chapitre, \(I\) désigne un intervalle non trivial de \(\mathbb{R}\) (c’est-à-dire non vide et non réduit à un point). On considère l’ensemble \(\mathscr F \left(I,\mathbb{R}\right)\) des fonctions définies sur \(I\) à valeurs dans \(\mathbb{R}\).

L’ensemble \(\mathscr F \left(I,\mathbb{R}\right)\)

(Opérations sur les fonctions).
Dans \(\mathscr F \left(I,\mathbb{R}\right)\), on définit les lois suivantes.
  • Addition. Si \((f,g) \in \mathscr F \left(I,\mathbb{R}\right)^2\), on définit l’application \(\left(f+g\right) \in \mathscr F \left(I,\mathbb{R}\right)\) par \[\forall x \in I, \quad \left(f+g\right)(x)=f(x)+g(x)\]

  • Multiplication par un réel. Si \((\lambda,f) \in \mathbb{R} \times \mathscr F \left(I,\mathbb{R}\right)\), on définit l’application \(\left(\lambda f\right) \in \mathscr F \left(I,\mathbb{R}\right)\) par \[\forall x \in I, \quad \left(\lambda f\right)(x)=\lambda f(x)\]

  • Multiplication de deux fonctions. Si \((f,g) \in \mathscr F \left(I,\mathbb{R}\right)^2\), on définit l’application \(\left( fg\right) \in \mathscr F \left(I,\mathbb{R}\right)\) par \[\forall x \in I, \quad \left(fg\right)(x)= f(x) g(x)\]

  • Valeur absolue d’une fonction. Si \(f \in \mathscr F \left(I,\mathbb{R}\right)\), on définit l’application \(\left|f\right| \in \mathscr F \left(I,\mathbb{R}\right)\) par \[\forall x \in I, \quad \left|f\right|(x)=\left| f(x)\right|\]

  • Maximum, Minimum de deux fonctions. Si \((f,g) \in \mathscr F \left(I,\mathbb{R}\right)^2\), on définit les deux applications \(\sup\left(f+g\right) \in \mathscr F \left(I,\mathbb{R}\right)\) et \(\inf\left(f+g\right) \in \mathscr F \left(I,\mathbb{R}\right)\) par \[\forall x \in I, \quad \sup\left(f,g\right)(x)=\max\left\{f(x),g(x)\right\}\] \[\forall x \in I, \quad \inf\left(f,g\right)(x)=\min\left\{f(x),g(x)\right\}\]

La relation d’ordre \(\leqslant\) sur \(\mathbb{R}\) s’étend naturellement à \(\mathscr F \left(I,\mathbb{R}\right)\) en posant, pour \((f,g) \in \mathscr F \left(I,\mathbb{R}\right)^2\) \[f \leqslant g \Longleftrightarrow\forall x \in I, \quad f(x) \leqslant g(x)\]
Soit \(f\in \mathscr F \left(I,\mathbb{R}\right)\). On a
  • \(\left|f\right|=\sup\left(f,-f\right)\)

  • \(\sup(f,g)=\dfrac{f+g+\left|f-g\right|}{2}\)

  • \(\inf(f,g)=\dfrac{f+g-\left|f-g\right|}{2}\)

En posant \(\left\{\begin{array}{l} f^+ = \sup \left(f,0\right) \\ f^- = \sup\left(-f,0\right) \end{array}\right.\), on vérifie que \(\left\{\begin{array}{l} f^+ = {\scriptstyle\left|f\right|+f\over\scriptstyle 2} \\ f^- = {\scriptstyle\left|f\right|-f\over\scriptstyle 2} \end{array}\right.\) et \(\left\{\begin{array}{l} f = f^+ - f^- \newline \left|f\right| = f^+ + f^- \end{array}\right.\)
  • \(\left(\mathscr F \left(I,\mathbb{R}\right),+,.\right)\) (où \(.\)  désigne la multiplication par un scalaire) possède une structure d’espace vectoriel sur \(\mathbb{R}\).

  • \(\left(\mathscr F \left(I,\mathbb{R}\right),+,\times\right)\) (où \(\times\) désigne le produit entre deux fonctions) possède une structure d’anneau.

  • L’élément neutre pour l’addition est la fonction identiquement nulle, \(0_{\mathcal{F}\left(I, \mathbb{R} \right)} : \left\{ \begin{array}{ccl} I & \longrightarrow & \mathbb{R} \\ x & \longmapsto & 0 \end{array} \right.\) et l’élément neutre pour la multiplication est la fonction constante \(1_{\mathcal{F}\left( , \mathbb{R} \right)}:\left\{ \begin{array}{ccl} I & \longrightarrow & \mathbb{R} \newline x & \longmapsto & 1 \end{array} \right.\).

Fonctions bornées

(Fonction majorée, minorée, bornée).
Soit \(f \in \mathscr F \left(I,\mathbb{R}\right)\). On dit que \(f\) est:
  • Majorée si et seulement si \(\exists M \in \mathbb{R}, \quad \forall x \in I, \quad f(x) \leqslant M\).

  • Minorée si et seulement si \(\exists m \in \mathbb{R}, \quad \forall x \in I, \quad f(x) \geqslant m\).

  • Bornée si elle est majorée et minorée.

(Pour montrer qu’une fonction est bornée sur \(I\), il suffit de la majorer, sur \(I\), en valeur absolue).
Une fonction \(f\in \mathscr F \left(I,\mathbb{R}\right)\) est bornée si et seulement si elle est majorée en valeur absolue, c’est-à-dire \[\exists \alpha \in \mathbb{R}~ \forall x\in I, \quad \left|f(x)\right| \leqslant\alpha .\]
  • Toute combinaison linéaire de fonctions bornées est bornée (l’ensemble des fonctions bornées forme un sous-espace vectoriel de \(\mathscr F \left(I,\mathbb{R}\right)\)).

  • Tout produit de deux fonctions bornées est encore borné.

  • Dire que \(f\in \mathscr F \left(I,\mathbb{R}\right)\) est majorée revient à dire que \(\left\{ f(x) ~ | ~ x \in I \right\}\) est un sous-ensemble majoré de \(\mathbb{R}\). Comme ce sous-ensemble est non vide, d’après l’axiome de la borne supérieure, il possède une borne supérieure qu’on note \(\displaystyle{\sup_{I} f}\).

  • De même, si \(f\in \mathscr F \left(I,\mathbb{R}\right)\) est minorée alors ce sous-ensemble est minoré. On note \(\displaystyle{\inf_I f}\) sa borne inférieure.

  • Si une fonction \(f\) est bornée, puisque \(\lvert f \rvert\) est majorée, la partie \(\{\lvert f(x) \rvert ;~x \in I\}\) possède une borne supérieure que l’on notera \(\displaystyle{\sup_{I} \lvert f \rvert } = \lVert f \rVert_{\infty}\).

(Extremum, Extremum local).
Soit \(f\in \mathscr F \left(I,\mathbb{R}\right)\) et soit \(a\in I\)
  • On dit que \(f\) admet un maximum en \(a\) si et seulement si \(\forall x\in I, \quad f(x) \leqslant f(a)\).

  • On dit que \(f\) admet un maximum local en \(a\) si et seulement si \(\exists h >0,~\forall x\in I,~ \left|x-a\right|\leqslant h \Rightarrow \quad f(x) \leqslant f(a)\).

  • On définit de manière analogue la notion de minimum et de minimum local.

  • On dit que \(f\) admet un extrémum (respectivement un extremum local) si \(f\) admet un maximum (respectivement un maximum local) ou un minimum (respectivement un minimum local).

Soit \(f\in \mathscr F \left(I,\mathbb{R}\right)\)
  • Si \(f\) possède un maximum sur \(I\), on le note \(\displaystyle{\max_I f}\)

  • De même, si \(f\) possède un minimum sur \(I\), on le note \(\displaystyle{\min_I f}\)

Monotonie

(Fonction croissante, décroissante, strictement croissante, ....).
Soit \(f \in \mathscr F \left(I,\mathbb{R}\right)\). On dit que :
  • \(f\) est croissante si et seulement si \(\forall x,y\in I,~ x\leqslant y \Rightarrow f(x)\leqslant f(y)\).

  • \(f\) est décroissante si et seulement si \(\forall x,y\in I,~x\leqslant y \Rightarrow f(x)\geqslant f(y)\).

  • \(f\) est monotone si et seulement si \(f\) est croissante ou décroissante.

On dit de plus que \(f\) est strictement croissante , strictement décroissante ou strictement monotone si et seulement si l’inégalité correspondante est stricte.
(Règle des signes).
Soient \(f:I \rightarrow \mathbb{R}\) et \(g: J \rightarrow \mathbb{R}\) toutes deux monotones et telles que \(f(I) \subset J\). On peut alors définir la fonction composée \(g\circ f: I \rightarrow \mathbb{R}\) qui est également monotone et l’on a la règle des signes pour la monotonie de \(g \circ f\).
\(f / g\) \(\nearrow\) \(\searrow\)
\(\nearrow\) \(\nearrow\) \(\searrow\)
\(\searrow\) \(\searrow\) \(\nearrow\)
Supposons par exemple \(f\) croissante sur \(I\) et \(g\) décroissante sur \(J\). Montrons que \(g\circ f\) est décroissante. Soient \((x, y) \in I\) tels que \(x \leqslant y\). Comme \(f\) est croissante, \(f(x) \leqslant f(y)\) et puisque \(g\) est décroissante, \(g\left(f(x)\right) \geqslant g\left(f(y)\right)\) et donc \(g\circ f(x) \geqslant g\circ f(x)\).
Soit \(f \in \mathscr F \left(I,\mathbb{R}\right)\) strictement monotone sur \(I\) et soit \(J=f(I)\) alors \(f\) réalise une bijection de \(I\) sur \(J\) et sa bijection réciproque \(f^{-1}:J \rightarrow I\) est strictement monotone de même sens que \(f\).
Supposons par exemple la fonction \(f\) strictement croissante sur \(I\). Montrons qu’alors \(f\) est injective. Soient \((x, y) \in I^2\) tels que \(f(x) = f(y)\), montrons que \(x = y\) par l’absurde. Si l’on avait \(x \neq y\), on aurait \(x < y\) ou \(y < x\), mais alors, puisque \(f\) est strictement croissante, on aurait \(f(x) < f(y)\) ou \(f(y) < f(x)\) ce qui est absurde. Puisque \(J=f(I)\), par définition de l’image directe d’une fonction, la fonction \(f\) est surjective de \(I\) vers \(J\). Elle réalise donc une bijection de \(I\) vers \(J\). Vérifions que la fonction \(f^{-1}\) est également strictement croissante. Soient \((X, Y) \in J^2\) tels que \(X < Y\). Notons \(x = f^{-1}(x)\) et \(y = f^{-1}(Y)\). Si l’on avait \(y \leqslant x\), puisque \(f\) est croissante, on aurait \(f(y) \leqslant f(x)\) et donc \(Y \leqslant X\) ce qui est faux. On en déduit que \(x < y\) donc que \(f^{-1}(X) < f^{-1}(Y)\).
Soit \(f\) une fonction bijective sur \(I\). Le graphe de \(f^{-1}\), dans un repère orthonormé, se déduit de celui de \(f\) par une symétrie d’axe la première bissectrice.

Parité périodicité

(Fonction paire, impaire).
Soit une fonction \(f\in \mathscr F \left(I,\mathbb{R}\right)\) . On suppose que l’intervalle \(I\) est symétrique par rapport à l’origine (c’est-à-dire que si \(x\in I\) alors \(-x\in I\)). On dit que
  • La fonction \(f\) est paire si et seulement si \(\forall x\in I,~ f(-x)=f(x)\)

  • La fonction \(f\) est impaire si et seulement si \(\forall x\in I,~f(-x)=-f(x)\).

Le graphe d’une fonction paire est symétrique par rapport à l’axe des ordonnées. Le graphe d’une fonction impaire est symétrique par rapport à l’origine du repère.
L’ensemble des fonctions paires (resp. impaires) est stable par combinaison linéaire. C’est un sous-espace vectoriel de \(\mathscr F \left(I,\mathbb{R}\right)\). Les sous-espaces de \(\mathscr F \left(I,\mathbb{R}\right)\) formés par les fonctions paires et par les fonctions impaires sont de plus supplémentaires dans \(\mathscr F \left(I,\mathbb{R}\right)\).
(Fonction périodique).
Une fonction \(f\) définie sur \(\mathbb{R}\) est périodique si et seulement s’il existe un réel \(T>0\) tel que \(\forall x\in I,~f(x+T)=f(x)\). On dit que \(T\) est une période de \(f\) et que \(f\) est \(T\)-périodique.
Soit \(T>0\). L’ensemble des fonctions \(T\)-périodiques sur \(\mathbb{R}\) est stable par combinaison linéaire et par produit. En particulier, c’est un sous-espace vectoriel de \(\mathscr F \left(I,\mathbb{R}\right)\).

Fonctions lipschitziennes

Rudolf Lipschitz, né le 14 mai 1832 à Königsberg, mort le 07 octobre 1903 à BonnMathématicien Allemand. Rudolf Lipschitz se caractérise par la grande diversité de ses contributions : fonctions de Bessel, séries de Fourier (il est à l’origine d’un critère pour tester leur convergence), géométrie Riemannienne, mécanique (il travailla à résoudre les équations du mouvement dans le formalisme d’Hamilton-Jacobi), théorie des nombres (il étudia les quaternions et, plus généralement, les algèbres de Clifford qu’il redécouvra et qu’il appliqua à la représentation des rotations d’un espace euclidien). Il est en particulier célèbre pour son amélioration du théorème de Cauchy quant à l’existence des solutions d’une équation différentielle. C’est lors de ce travail qu’il introduisit les fonctions qui maintenant portent son nom et que nous allons étudier dans ce paragraphe.
image

(Fonctions lipschitziennes).
  Soit un réel \(k \geqslant 0\).
  • On dit qu’une fonction \(f: I \mapsto \mathbb{R}\) est \(k\)-lipschitzienne sur l’intervalle \(I\) si et seulement si \[\boxed{\forall (x,y)\in I^2,\quad \left|f(x)-f(y)\right|\leqslant k\lvert x-y \rvert }\]

  • On dit qu’une fonction est lipschitzienne sur l’intervalle \(I\) s’il existe \(k\geqslant 0\) telle que \(f\) soit \(k\)-lipschitzienne.

  • On note \(\mathscr{L}(I)\) l’ensemble des fonctions lipschitziennes sur l’intervalle \(I\).

On comprend mieux cette définition en écrivant la propriété équivalente, \[\exists k \geqslant 0,~ \forall (x,y) \in I^2,~ x \neq y,~ \left|\dfrac{f(x)-f(y)}{x-y}\right| \leqslant k\] Une fonction est lipschitzienne sur l’intervalle \(I\) si et seulement si l’ensemble des pentes de toutes ses cordes est borné.
(Opérations sur les fonctions lipschitziennes).
  1. Une combinaison linéaire de deux fonctions lipschitzienne est encore lipschitzienne. Si \(f, g \in \mathscr{L}(I)\), alors \(\alpha~f~+~\beta~g~\in~\mathscr{L}(I)\).

  2. La composée de deux fonctions lipschitziennes est encore lipschitzienne. Si \(f \in \mathscr{L}(I)\) et \(g \in \mathscr{L}(J)\) avec \(f(I) \subset J\), alors \(g\circ f \in \mathscr{L}(I)\).

  3. Soit \(c \in I\), on note \(I_1 = I \cap ]-\infty,c]\) et \(I_2 = I \cap [c, +\infty[\). Si \(f\) est lipschitzienne sur \(I_1\) et sur \(I_2\), alors elle est lipschitzienne sur \(I\).

  1. Puisque \(f\) et \(g\) sont lipschitziennes sur \(I\), il existe deux constantes \(k_1, k_2 \geqslant 0\) telles que \(\forall (x,y) \in I^2\), \(\lvert f(x)-f(y) \rvert \leqslant k_1 \lvert x-y \rvert\) et \(\lvert g(x)-g(y) \rvert \leqslant k_2 \lvert x-y \rvert\). Posons \(k = \lvert \alpha \rvert k_1 + \lvert \beta \rvert k_2\) et vérifions que \(\alpha f + \beta g\) est \(k\)-lipschitzienne. Soit \((x,y) \in I^2\), utilisons l’inégalité triangulaire \[\lvert (\alpha f + \beta g)(x) - (\alpha f + \beta g)(y) \rvert = \left|\alpha\left(f(x)-f(y)\right) + \beta\left(g(x)-g(y)\right)\right| \leqslant\lvert \alpha \rvert \lvert f(x)-f(y) \rvert + \lvert \beta \rvert \lvert g(x)-g(y) \rvert \leqslant(\lvert \alpha \rvert k_1 + \lvert \beta \rvert k_2)\lvert x-y \rvert\]

  2. Comme \(f\) est lipschitzienne sur \(I\), il existe \(k_1 \geqslant 0\) tel que \(\forall (x,y) \in I^2\), \(\lvert f(x)-f(y) \rvert \leqslant k_1 \lvert x-y \rvert\). Puisque \(g\) est lipschitzienne sur \(J\), il existe \(k_2 \geqslant 0\) tel que \(\forall (X,Y) \in J^2\), \(\lvert g(X)-g(Y) \rvert \leqslant k_2 \lvert X-Y \rvert\). Posons \(k = k_1k_2\) et vérifions que \(g\circ f\) est \(k\)-lipschitzienne sur \(I\). Soient \((x,y)\in I^2\), puisque \(X = f(x)\in J\) et \(Y=f(y)\in J\), \[\lvert g\circ f(x)-g\circ f(y) \rvert = \lvert g(X)-g(Y) \rvert \leqslant k_2 \lvert X-Y \rvert = k_2\lvert f(x)-f(y) \rvert \leqslant k_1k_2 \lvert x-y \rvert\]

  3. Puisque \(f\) est lipschitzienne sur \(I_1\), il existe \(k_1 \geqslant 0\) tel que \(\forall (x,y) \in I_1^2\), \(\lvert f(x)-f(y) \rvert \leqslant k_1 \lvert x-y \rvert\) et puisque \(f\) est lipschitzienne sur \(I_2\), il existe \(k_2 \geqslant 0\) tel que \(\forall (x,y) \in I_2^2\), \(\lvert f(x)-f(y) \rvert \leqslant k_2 \lvert x-y \rvert\). Posons \(k = \max(k_1,k_2)\) et vérifions que \(f\) est \(k\)-lipschitzienne sur \(I\). Soient \((x,y) \in I^2\). Étudions trois cas ,

    • Si \(x,y \in I_1\), alors \(\lvert f(x)-f(y) \rvert \leqslant k_1\lvert x-y \rvert \leqslant k \lvert x-y \rvert\).

    • Si \(x,y \in I_2\), alors \(\lvert f(x)-f(y) \rvert \leqslant k_2\lvert x-y \rvert \leqslant k \lvert x-y \rvert\).

    • Si \(x \in I_1\) et \(y \in I_2\) (l’autre cas est similaire), puisque \((x,c) \in I_1^2\) et \((c,y) \in I_2^2\) et que \(x < c < y\), \[\begin{aligned} \lvert f(x)-f(y) \rvert &= \left|\left[f(x)-f(c)\right] + \left[f(c)-f(y)\right]\right| \leqslant \lvert f(x)-f(c) \rvert \leqslant k_1 \lvert c-x \rvert + k_2\lvert y-c \rvert \newline &= k_1(c-x)+k_2(y-c) \leqslant k(c-x)+k(y-c)= k(y-x) = k\lvert y-x \rvert \end{aligned}\]

Limite et continuité en un point

Voisinage

(Point adhérent).
Soit \(A \subset \mathbb{R}\) une partie de \(\mathbb{R}\). On dit qu’un réel \(x\) est adhérent à la partie \(A\) lorsque \(\forall \varepsilon> 0\), \(\exists a \in A\) tel que \(\lvert x-a \rvert \leqslant\varepsilon\). On note \(\overline{A}\) l’ensemble des points adhérents de la partie \(A\).
On dira également que \(+\infty\) est un point adhérent à la partie \(A\) lorsque \(\forall M > 0\), \(\exists a \in A\) tel que \(a \geqslant M\) et que \(-\infty\) est adhérent à la partie \(A\) lorsque \(\forall m < 0\), \(\exists a \in A\) tel que \(a \leqslant m\).
(Voisinage d’un point).
Soit \(V\) une partie de \(\mathbb{R}\) et un point adhérent \(a \in \overline{V}\). On dit que
  • \(V\) est un voisinage de \(a\) si et seulement si il existe \(\varepsilon>0\) tel que \(\left]a-\varepsilon,a+\varepsilon\right[ \subset V\).

  • \(V\) est un voisinage de \(+\infty\) si et seulement si il existe \(B\in\mathbb{R}\) tel que \(\left]B,+\infty\right[ \subset V\).

  • \(V\) est un voisinage de \(-\infty\) si et seulement si il existe \(A\in\mathbb{R}\) tel que \(\left]-\infty,A\right[ \subset V\).

On note \(V_a\) l’ensemble des voisinages du point \(a\).
(Propriété vraie au voisinage d’un point).
Soient \(f\) une fonction définie sur une partie \(I\) de \(\mathbb{R}\) et \(a\in \overline{I}\).
  • On dit que la fonction \(f\) est définie au voisinage du point \(a\) si et seulement s’il existe un voisinage \(V\) de \(a\) telle que \(V\subset I\).

  • On dit que \(f\) vérifie la propriété \(\mathcal{P}\) au voisinage du point \(a\) si et seulement s’il existe un voisinage \(V\subset I\) de \(a\) tel que la restriction de \(f\) à \(V\) vérifie la propriété \(\mathcal{P}\).

Notion de limite

(Limite d’une fonction en un point).
Soient une fonction \(f\in \mathscr F \left(I,\mathbb{R}\right)\), un point adhérent \(a\in \overline I\) et un réel \(\ell \in \mathbb{R}\). On dit que la fonction \(f\) admet pour limite le réel \(\ell\) en \(a\) lorsque
  • Si \(a\in \mathbb{R}\) : \(\forall \varepsilon >0,~\exists \eta >0,~\forall x \in I,~\left|x-a\right|\leqslant\eta \Rightarrow \left|f(x)-\ell\right| \leqslant\varepsilon\).

  • Si \(a=+\infty\) : \(\forall \varepsilon >0,~\exists M \in \mathbb{R},~\forall x \in I,~x \geqslant M \Rightarrow \left|f(x)-\ell\right| \leqslant\varepsilon\).

  • Si \(a=-\infty\) : \(\forall \varepsilon >0,~\exists m \in \mathbb{R},~\forall x \in I,~x \leqslant m \Rightarrow \left|f(x)-\ell\right| \leqslant\varepsilon\).

On note alors \(f(x) \xrightarrow[x \rightarrow a]{} \ell\).
(Pour montrer que \(f(x) \xrightarrow[x \rightarrow a]{} \ell\)).
on utilise le plan
  1. Soit \(\varepsilon> 0\).

  2. Posons \(\eta = \dots > 0\).

  3. Vérifions : soit \(x \in I\) tel que \(\lvert x-a \rvert \leqslant\eta\) …on a bien \(\lvert f(x)-\ell \rvert \leqslant\varepsilon\).

Comme pour les suites, on peut utiliser des inégalités strictes \(\lvert f(x)-\ell \rvert < \varepsilon\), \(\lvert x-a \rvert < \eta\), \(x > M\) …dans ces définitions.
image

Avec \(\varepsilon=0.2\)

image

Avec \(\varepsilon=0.2\)

(Unicité de la limite).
Si \(f\) admet pour limites en \(a \in \overline{I}\) les réels \(\ell\) et \(\ell'\) alors \(\ell=\ell'\). On dira que \(\ell\) est la limite de \(f\) en \(a\) et on écrira \(\ell= \displaystyle{\lim_{x \rightarrow a}f(x)}\) ou \(\displaystyle{\ell = \lim_{a} f}\).
Démontrons le résultat par exemple lorsque \(a \in \mathbb{R}\). Supposons par l’absurde que \(\ell \neq \ell'\), et posons \(\varepsilon= \lvert \ell'-\ell \rvert /2 > 0\). Puisque \(f(x) \xrightarrow[x \rightarrow a]{} l\), il existe \(\eta_1 >0\) tel que \(\forall x \in I\), \(\lvert x-a \rvert \leqslant\eta_1 \Rightarrow \lvert f(x)-\ell \rvert < \varepsilon\). De même, puisque \(f(x) \xrightarrow[x \rightarrow a]{} \ell'\), il existe \(\eta_2 > 0\) tel que \(\forall x \in I\), \(\lvert x-a \rvert \leqslant\eta_2 \Rightarrow \lvert f(x)-\ell' \rvert < \varepsilon\). Posons \(\eta = \min(\eta_1, \eta_2) > 0\). Puisque \(a\) est adhérent à \(I\), il existe \(x \in I\) tel que \(\lvert x-a \rvert \leqslant\eta\) mais alors \(\lvert f(x)-\ell \rvert < \varepsilon\) et \(\lvert f(x)-\ell' \rvert < \varepsilon\) et donc \[\lvert \ell-\ell' \rvert = \lvert (\ell-f(x)) + (f(x)-\ell) \rvert \leqslant\lvert f(x)-\ell \rvert + \lvert f(x)-\ell' \rvert < 2\varepsilon= \lvert \ell'-\ell \rvert\] ce qui est absurde.
(Limite infinie).
Soit une fonction \(f\in \mathscr F \left(I,\mathbb{R}\right)\) et un point adhérent \(a \in I\). On dit que la fonction \(f\) tend vers \(+\infty\) (respectivement \(-\infty\)) lorsque \(x\) tend vers \(a\) lorsque
  • Si \(a\in \mathbb{R}\) : \(\forall B\in \mathbb{R},~\exists \eta >0,~\forall x \in I,~\left|x-a\right|\leqslant\eta \Rightarrow f(x) \geqslant B\) (respectivement \(f(x) \leqslant B\)).

  • Si \(a=+\infty\) : \(\forall B\in \mathbb{R},~\exists A \in \mathbb{R}~\forall x \in I,~x \geqslant A \Rightarrow f(x) \geqslant B\) (respectivement \(f(x) \leqslant B\)).

  • Si \(a=-\infty\) : \(\forall B\in \mathbb{R},~\exists A \in \mathbb{R}~\forall x \in I,~x \leqslant A \Rightarrow f(x) \geqslant B\) (respectivement \(f(x) \leqslant B\)).

On notera alors \(f(x) \xrightarrow[x \rightarrow a]{} +\infty\) (respectivement \(f(x)\xrightarrow[x \rightarrow a]{} -\infty\)).
image

\(f(x) \xrightarrow[x\rightarrow a^+]{}-\infty\)

image

\(f(x) \xrightarrow[x\rightarrow a^+]{}-\infty\)

(Pour montrer que \(f(x) \xrightarrow[x \rightarrow a]{} +\infty\)).
Dans le cas où \(a\) est fini, on utilise le plan
  1. Soit \(B \in \mathbb{R}\).

  2. Posons \(\eta = \dots > 0\).

  3. Soit \(x \in I\) tel que \(\lvert x-a \rvert \leqslant\eta\).

  4. On a bien \(f(x) \geqslant B\).

Pour montrer que \(f(x) \xrightarrow[x \rightarrow +\infty]{} -\infty\),

  1. Soit \(B \in \mathbb{R}\).

  2. Posons \(A = \dots\)

  3. Soit \(x \in I\) tel que \(x \geqslant A\).

  4. On a bien \(f(x) \leqslant B\).

Voici une formulation équivalente de la notion de limite (finie et infinie) qui ne fait intervenir que la notion de voisinages.

(Définition de la limite à l’aide des voisinages).
Soient \(f\in \mathscr F \left(I,\mathbb{R}\right)\), \(a\in\overline I\) et \(l\in\overline{\mathbb{R}}\). \[f(x) \xrightarrow[x \rightarrow a]{} l \Longleftrightarrow \forall W\in V_{l},~\exists V \in V_a,~f\left(V\cap I\right)\subset W\]
Démontrons le résultat dans le cas où \(a\) et \(l\) sont finis.
  • Soit \(W\) un voisinage de \(l\), il existe \(\varepsilon> 0\) tel que \(]l-\varepsilon, l+\varepsilon[ \subset W\). Puisque \(f(x) \xrightarrow[x \rightarrow a]{} l\), il existe \(\eta > 0\) tel que \(\forall x \in I\), \(\lvert x-a \rvert \leqslant\eta \Rightarrow \lvert f(x)-l \rvert < \varepsilon\). Posons donc \(V = ]a-\eta,a+\eta[\) qui est un voisinage du point \(a\). Soit \(y \in f(V\cap I)\), il existe \(x \in V\cap I\) tel que \(y = f(x)\) et puisque \(x \in ]a-\eta, a+\eta[ \cap I\), on a \(\lvert f(x)-l \rvert < \varepsilon\) et donc \(y=f(x) \in W\).

  • Soit \(\varepsilon> 0\). Posons \(W = ]l-\varepsilon, l+\varepsilon[\) qui est un voisinage du point \(l\). Il existe donc un voisinage \(V\) du point \(a\) tel que \(f(V\cap I) \subset W\). D’après la définition d’un voisinage, il existe \(\eta > 0\) tel que \(]a-\eta, a+\eta[ \subset V\). Soit alors \(x \in I\) tel que \(\lvert x-a \rvert < \eta\), puisque \(x \in V\cap I\), \(f(x) \in W\) d’où \(\lvert f(x)-l \rvert < \varepsilon\).

(Limite finie \(\Rightarrow\) localement bornée).
Soit \(f\in \mathscr F \left(I,\mathbb{R}\right)\) une fonction admettant une limite finie en \(a\in \overline I\). Alors il existe un voisinage \(V\) du point \(a\) sur lequel la fonction \(f\) est bornée. \(a\).
Démontrons le résultat lorsque \(a \in \mathbb{R}\). Prenons \(\varepsilon= 1\) dans la définition de la limite, il existe \(\eta > 0\) tel que \(\forall x \in I\), \(\lvert x-a \rvert \leqslant\eta \Rightarrow \lvert f(x)-l \rvert \leqslant 1\). Notons \(V = ]a-\eta, a+\eta[\) qui est un voisinage du point \(a\) et \(M = \lvert l \rvert + 1\). Pour \(x \in V \cap I\), d’après la minoration de l’inégalité triangulaire, \(\lvert f(x) \rvert - \lvert l \rvert \leqslant\lvert f(x)-l \rvert \leqslant 1\) d’où \(\lvert f(x) \rvert \leqslant M\).
(Transformation de limite en inégalité).
Soit \(f \in \mathscr F \left(I,\mathbb{R}\right)\) une fonction, \(a \in \overline{I}\) et \(l \in \mathbb{R}\) et deux réels \(k,k' \in \mathbb{R}\). On suppose que
  1. \(f(x) \xrightarrow[x \rightarrow a]{} l\).

  2. \(k < l < k'\).

Alors il existe un voisinage \(V\) du point \(a\) tel que \(\forall x \in V\cap I\), \(k \leqslant f(x) \leqslant k'\).
Posons \(\varepsilon= \min(l-k, k'-l)\). Puisque \(f(x) \xrightarrow[x \rightarrow a]{} l\), il existe un voisinage \(V\) du point \(a\) tel que \(\forall x \in V \cap I\), \(\lvert f(x) -l \rvert \leqslant\varepsilon\) d’où si \(x \in V\cap I\), \(f(x) - l \leqslant\varepsilon\) ce qui donne \(f(x) \leqslant l + \varepsilon\leqslant l + (k'-l) \leqslant k'\) et aussi \(l-f(x) \leqslant\varepsilon\) ce qui donne \(f(x) \geqslant l - \varepsilon\geqslant k\).

Pour montrer qu’une fonction tend vers \(l\) lorsque \(x\) tend vers \(a\), on majore en pratique \(\lvert f(x)-l \rvert\) par une fonction qui tend vers zéro.

(Théorème de majoration).
Soient
  • une fonction \(f:I \rightarrow \mathbb{R}\), \(a\in \bar{I}\) et \(l\in \mathbb{R}\).

  • \(\theta\) une fonction définie sur un voisinage \(V\) de \(a\)

On suppose que

  1. \(\forall x\in V, \quad \left|f(x) - l\right| \leqslant\theta \left(x\right)\).

  2. \(\theta \left(x\right) \xrightarrow[x\rightarrow a]{} 0\)

alors \(f(x) \xrightarrow[x\rightarrow a]{} l\).
Remarquons qu’en vertu de l’inégalité énoncée dans la première hypothèse, \(\theta\) est nécessairement positive. Soit \(\varepsilon>0\). Comme \(\theta \left(x\right) \xrightarrow[x\rightarrow a]{} 0\), il existe un voisinage \(V_2\) de \(a\) tel que : \(\forall x\in V,\quad \left|\theta\left(x\right)\right| = \theta\left(x\right)\leqslant\varepsilon\). Soit \(x\in V\), en appliquant la première hypothèse : \(\left|f\left(x\right)-l\right|\leqslant \theta\left(x\right)\leqslant\varepsilon\). Ce qui prouve que \(f(x) \xrightarrow[x\rightarrow a]{} l\).

Opérations algébriques sur les limites

Les démonstrations de ce paragraphe sont typiques des démonstrations à \(\varepsilon\) d’analyse. Il est important de les étudier en détail et de les comparer aux démonstrations correspondantes sur les suites.

(Limite d’une somme).
Soient \(f, g : I \mapsto \mathbb{R}\) deux fonctions et \(a \in \overline{I}\). On suppose que \(f(x) \xrightarrow[x \rightarrow a]{} l_1\) et que \(g(x) \xrightarrow[x \rightarrow a]{} l_2\). Alors, \((f+g)(x)~\xrightarrow[x \rightarrow a]{}~l_1~+~l_2\).
(1).
Notre hypothèse permet de majorer \(\lvert f(x)-l_1 \rvert\) et \(\lvert g(x)-l_2 \rvert\) par \(\varepsilon'\) aussi petit que l’on veut pour \(x\) suffisamment proche de \(a\). Faisons apparaître cette quantité sous la valeur absolue avant de majorer à l’aide de l’inégalité triangulaire : \[\lvert (f+g)(x)-(l_1+l_2) \rvert = \left|\left(f(x)-l_1\right) + \left(g(x)-l_2\right)\right| \leqslant \lvert f(x)-l_1 \rvert + \lvert g(x)-l_2 \rvert \leqslant 2\varepsilon'\] Il nous reste à rédiger une démonstration rigoureuse en suivant le plan de démonstration correspondant à la définition de la limite.
  1. Soit \(\varepsilon> 0\).

  2. Posons \(\varepsilon' = \varepsilon/2\). Puisque \(f(x) \xrightarrow[x \rightarrow a]{} l_1\), il existe \(\eta_1 > 0\) tel que \(\forall x \in I\), \(\lvert x-a \rvert \leqslant\eta_1 \Rightarrow \lvert f(x)-l_1 \rvert \leqslant\varepsilon'\). De même, il existe \(\eta_2 > 0\) tel que \(\forall x \in I\), \(\lvert x-a \rvert \leqslant\eta_2 \Rightarrow \lvert g(x)-l_2 \rvert \leqslant\varepsilon'\).

  3. Posons \(\eta = \min(\eta_1, \eta_2)\).

  4. Soit \(x \in I\) tel que \(\lvert x-a \rvert \leqslant\eta\), on a bien \[\lvert (f+g)(x)-(l_1+l_2) \rvert = \left|\left(f(x)-l_1\right) + \left(g(x)-l_2\right)\right| \leqslant \lvert f(x)-l_1 \rvert + \lvert g(x)-l_2 \rvert \leqslant\varepsilon' + \varepsilon' = \varepsilon\]

On montre de même que pour tous réels \(\alpha, \beta \in \mathbb{R}\), la fonction combinaison linéaire \(\alpha f + \beta g\) tend vers la combinaison linéaire des limites : \((\alpha f + \beta g)(x) \xrightarrow[x \rightarrow a]{} \alpha l_1 + \beta l_2\).
(Limite d’un produit).
Soient \(f, g : I \mapsto \mathbb{R}\) et \(a \in \overline{I}\). On suppose que \(f(x) \xrightarrow[x \rightarrow a]{} l_1\), \(g(x) \xrightarrow[x \rightarrow a]{} l_2\). Alors \((fg)(x) \xrightarrow[x \rightarrow a]{} l_1l_2\).
(1).
Estimons la différence en faisant apparaître notre hypothèse \(\lvert f(x)-l_1 \rvert \leqslant\varepsilon'\) et \(\lvert g(x)-l_2 \rvert \leqslant\varepsilon'\). \[\lvert (fg)(x) - l_1l_2 \rvert = \left| f(x)\left[g(x)-l_2\right] + l_2\left[f(x)-l_1\right]\right| \leqslant\lvert f(x) \rvert \lvert g(x)-l_2 \rvert + \lvert l_2 \rvert \lvert f(x)-l_1 \rvert\] Il reste \(\lvert f(x) \rvert\) que l’on peut majorer puisque \(f\) est bornée sur un voisinage de \(a\). Maintenant que nous avons compris pourquoi le résultat est vrai, écrivons une preuve rigoureuse qui utilise le plan de démonstration de limite.
  1. Soit \(\varepsilon> 0\).

  2. Comme \(f\) admet une limite finie au point \(a\), elle est bornée sur un voisinage de \(a\) donc il existe \(\eta_3 > 0\) et \(M>0\) tel que \(\forall x \in I\), \(\lvert x-a \rvert \leqslant\eta_3 \Rightarrow \lvert f(x) \rvert \leqslant M\).

  3. Notons \(\varepsilon' = \varepsilon/(\lvert l_2 \rvert + M)\). Puisque \(f(x) \xrightarrow[x \rightarrow a]{} l_1\), il existe \(\eta_1 > 0\) tel que \(\forall x \in I\), \(\lvert x-a \rvert \leqslant\eta_1 \Rightarrow \lvert f(x)-l_1 \rvert \leqslant\varepsilon'\). Puisque \(g(x) \xrightarrow[x \rightarrow a]{} l_2\), il existe \(\eta_2 > 0\) tel que \(\forall x \in I\), \(\lvert x-a \rvert \leqslant\eta_2 \Rightarrow \lvert g(x)-l_2 \rvert \leqslant\varepsilon'\).

  4. Posons \(\eta = \min(\eta_1,\eta_2,\eta_3) > 0\).

  5. Soit \(x \in I\) tel que \(\lvert x-a \rvert \leqslant\eta\), en recopiant la majoration précédente, \[\lvert (fg)(x)-l_1l_2 \rvert \leqslant\lvert f(x) \rvert \lvert g(x)-l_2 \rvert + \lvert l_2 \rvert \lvert f(x)-l_1 \rvert \leqslant(M+\lvert l_2 \rvert )\varepsilon' = \varepsilon\]

Remarquez l’ordre dans lequel les différents objets sont introduits dans la démonstration. Pour définir \(\varepsilon'\), il fallait avoir défini \(M\) auparavant.
(Limite d’une valeur absolue).
Soit \(f : I \mapsto \mathbb{R}\), \(a \in \overline{I}\) et \(l \in \mathbb{R}\). Si \(f(x) \xrightarrow[x \rightarrow a]{} l\), alors \(\lvert f \rvert (x) \xrightarrow[x \rightarrow a]{} \lvert l \rvert\).
(1).
  1. Soit \(\varepsilon> 0\).

  2. Puisque \(f(x) \xrightarrow[x \rightarrow a]{} l\), il existe \(\eta > 0\) tel que \(\forall x \in I\), \(\lvert x-a \rvert \leqslant\eta \Rightarrow \lvert f(x)-l \rvert \leqslant\varepsilon\).

  3. Soit \(x \in I\) tel que \(\lvert x-a \rvert \leqslant\eta\), grâce à la minoration de l’inégalité triangulaire, \[\left| \lvert f(x) \rvert - \lvert l \rvert \right| \leqslant\lvert f(x)-l \rvert \leqslant\varepsilon\]

(Limite de l’inverse).
Soit \(f : I \mapsto \mathbb{R}\), \(a \in \overline{I}\) et \(l \in \mathbb{R}\). On suppose que
  1. \(f(x) \xrightarrow[x \rightarrow a]{} l\).

  2. \(l \neq 0\).

Alors \((1/f)(x) \xrightarrow[x \rightarrow a]{} 1/l\).
(1).
Nous savons majorer \(\lvert f(x)-l \rvert\) par un réel \(\varepsilon'> 0\) aussi petit que l’on veut sur un voisinage de \(a\). Estimons la quantité \[\left|\dfrac{1}{f(x)} - \dfrac{1}{l} \right| = \dfrac{\lvert f(x)-l \rvert }{\lvert f(x) \rvert \lvert l \rvert }\] Il nous faut minorer \(\lvert f(x) \rvert\) au dénominateur. Puisque \(f(x) \xrightarrow[x \rightarrow a]{} l\), on a vu que \(\lvert f(x) \rvert \xrightarrow[x \rightarrow a]{} \lvert l \rvert\) et puisque \(\lvert l \rvert > 0\), en notant \(k = \lvert l \rvert /2\), puisque \(k < \lvert l \rvert\), d’après la proposition [thm:0718145314], il existe un voisinage de \(a\) sur lequel \(\lvert f(x) \rvert \geqslant k\) et alors \(\left|1/f(x)-1/l\right| \leqslant\varepsilon'/k\lvert l \rvert\). Rédigeons maintenant une preuve rigoureuse.
  1. Soit \(\varepsilon> 0\).

  2. Notons \(k = \lvert l \rvert /2\). Puisque \(l\neq 0\), \(k < \lvert l \rvert\) et comme \(\lvert f \rvert (x) \xrightarrow[x \rightarrow a]{} \lvert l \rvert\), d’après la transformation d’une limite en inégalité, il existe \(\eta_1 > 0\) tel que \(\forall x \in I\), \(\lvert x-a \rvert \leqslant\eta_1 \Rightarrow k < \lvert f(x) \rvert\).

  3. Notons \(\varepsilon' = k\lvert l \rvert > 0\). Puisque \(f(x) \xrightarrow[x \rightarrow a]{} l\), il existe \(\eta_2 > 0\) tel que \(\forall x \in I\), \(\lvert x-a \rvert \leqslant\eta_2 \Rightarrow \lvert f(x)-l \rvert \leqslant\varepsilon'\).

  4. Posons \(\eta = \min(\eta_1, \eta_2)\).

  5. Soit \(x \in I\) tel que \(\lvert x-a \rvert \leqslant\eta\), \[\left|\dfrac{1}{f(x)} - \dfrac{1}{l} \right| = \dfrac{\lvert f(x)-l \rvert }{\lvert f(x) \rvert \lvert l \rvert } \leqslant\dfrac{\varepsilon'}{k\lvert l \rvert } = \varepsilon\]

D’après les deux théorèmes précédents, si \(f(x) \xrightarrow[x \rightarrow a]{} l_1\) et \(g(x) \xrightarrow[x \rightarrow a]{} l_2\) avec \(l_2 \neq 0\), \((f/g)(x) \xrightarrow[x \rightarrow a]{} l_1/l_2\). On invoque souvent les théorèmes de ce paragraphe pour justifier l’existence d’une limite sous le nom de théorèmes généraux  sur les limites.

On peut étendre les théorèmes généraux aux limites infinies. Soient \(f, g : I \mapsto \mathbb{R}\) deux fonctions, \(a \in \overline{I}\), éventuellement infini et un réel \(\alpha\). On suppose que \(f(x) \xrightarrow[x \rightarrow a]{} l \in \overline{\mathbb{R}}\) et \(g(x) \xrightarrow[x \rightarrow a]{} l' \in \overline{\mathbb{R}}\). Nous avons résumé dans les tableaux suivants les limites de la somme, produit et quotient des deux fonctions dans tous les cas de figure. Les cases noires correspondent à des formes indéterminées  où l’on ne peut rien dire de général.

  • Somme \(\boxed{f+g}\)

     \(l'\) \(-\infty\) \(l'\in\mathbb{R}\) \(+\infty\)
    \(-\infty\) \(-\infty\) \(-\infty\)
    \(l\in\mathbb{R}\) \(-\infty\) \(l+l'\) \(+\infty\)
    \(+\infty\) \(+\infty\) \(+\infty\)
  • Produit \(\boxed{fg}\)

     \(l'\) \(-\infty\) \(l'<0\) \(l'=0\) \(l'>0\) \(+\infty\)
    \(-\infty\) \(+\infty\) \(+\infty\) \(-\infty\) \(-\infty\)
    \(l<0\) \(+\infty\) \(l l'\) \(0\) \(l l'\) \(-\infty\)
    \(l=0\) \(0\) \(0\) \(0\)
    \(l>0\) \(-\infty\) \(l l'\) \(0\) \(l l'\) \(+\infty\)
    \(+\infty\) \(-\infty\) \(-\infty\) \(+\infty\) \(+\infty\)
  • Inverse \(\boxed{\dfrac{1}{f}}\)

    \(l\) \(-\infty\) \(l<0\) \(l=0^{-}\) \(l=0\) \(l=0^{+}\) \(l>0\) \(+\infty\)
    \(\dfrac{1}{f}\) \(0\) \(1/l\) \(-\infty\) \(+\infty\) \(1/l\) \(0\)

Continuité

(Continuité en un point).
Soit \(f\) une fonction définie sur \(I\) et \(a\in I\). On dit que la fonction \(f\) est continue au point \(a\) si et seulement si \(f(x) \xrightarrow[x\rightarrow a]{} f(a)\) ce qui se traduit avec des quantificateurs par : \[\forall \varepsilon>0,~\exists \eta>0,~\forall x\in I,~\left|x-a\right|\leqslant\eta \Rightarrow \left|f\left(x\right)-f\left(a\right)\right|\leqslant\varepsilon\]
(Théorèmes généraux).
Soient \(f, g : I \mapsto \mathbb{R}\) deux fonctions continues en un point \(a \in I\), alors
  1. la fonction \((f+g)\) est continue au point \(a\),

  2. la fonction \((fg)\) est continue au point \(a\),

  3. si \(g(a) \neq 0\), la fonction \(f/g\) est définie sur un voisinage du point \(a\) est est continue au point \(a\).

\((1)\) et \((2)\) sont une conséquence directe des théorèmes généraux sur les limites. Vérifions \((3)\). Puisque \(\lvert g(a) \rvert \neq 0\) et que \(g\) est continue au point \(a\), \(g(x) \xrightarrow[x \rightarrow a]{} g(a)\) donc \(\lvert g(x) \rvert \xrightarrow[x \rightarrow a]{} \lvert g(a) \rvert\). Posons \(k = \lvert g(a) \rvert /2\), on a \(0 < k < \lvert g(a) \rvert\) donc d’après le théorème [thm:0718145314], il existe un voisinage \(V\) du point \(a\) tel que \(\forall x \in I \cap V\), \(0 < \lvert g(a)/2 \rvert < \lvert g(x) \rvert\) et donc la fonction \(g\) ne s’annule pas sur \(V\). La fonction \(f/g\) est donc définie sur \(I\cap V\) et d’après les théorèmes généraux sur les limites, \((f/g)(x) \xrightarrow[x \rightarrow a]{} (f/g)(a)\).

Limite à gauche, à droite, continuité à gauche, à droite

(Voisinages à gauche, à droite).
Soit \(a\in\mathbb{R}\). On dit qu’une partie \(V\) de \(\mathbb{R}\) est
  • un voisinage à droite de \(a\) lorsqu’il existe \(\varepsilon>0\) tel que \(\left[a,a+\varepsilon\right] \subset V\),

  • un voisinage à gauche de \(a\) lorsqu’il existe \(\varepsilon>0\) tel que \(\left[a-\varepsilon,a\right] \subset V\),

  • un voisinage strict à droite de \(a\) lorsqu’il existe \(\varepsilon>0\) tel que \(\left]a,a+\varepsilon\right] \subset V\),

  • un voisinage strict à gauche de \(a\) lorsqu’il existe \(\varepsilon>0\) tel que \(\left[a-\varepsilon,a\right[ \subset V\),

  • un voisinage pointé de \(a\) lorsqu’il existe \(\varepsilon>0\) tel que \(\left[a-\varepsilon,a\right[\cup\left]a,a+\varepsilon\right] \subset V\).

(Limite à gauche, à droite).
Soit une fonction \(f: I \setminus \left\{ a \right\} \rightarrow \mathbb{R}\). On dit qu’un réel \(l\) est la limite à droite (resp. à gauche) de \(f\) si il existe un voisinage strict à droite de \(a\) (resp. un voisinage strict à gauche de \(a\)) tel que la restriction de \(f\) à ce voisinage admet \(l\) pour limite en \(a\). Lorqu’elle existe, la limite à droite de \(f\) est unique et est notée \(l= \displaystyle{\lim_{x \rightarrow a^+}f(x)}\) ou \(l= \displaystyle{l = \lim_{\substack{{x}\rightarrow a \newlinex \geqslant a}} f(x)}\) ou \(\displaystyle{l = \lim_{a^+} f}\). Nous noterons également \(f(x) \xrightarrow[x\rightarrow a^+]{} l\). On a des notations identiques pour la limite à gauche.
En termes de quantificateurs, \(f\) admet \(l\in \mathbb{R}\) comme limite à gauche en \(a\) si et seulement si \[\forall \varepsilon >0,~\exists \eta >0,~\forall x \in I,~a-\eta \leqslant x < a \Rightarrow \left|f(x)-l\right| \leqslant\varepsilon\]
Une fonction \(f\) possède une limite en \(a\) lorsque
  • \(f\) admet une limite à gauche \(l_1\in \mathbb{R}\).

  • \(f\) admet une limite à droite \(l_2\in \mathbb{R}\).

  • \(l_1=l_2\).

(Continuité à gauche, à droite).
Soient \(f: I \rightarrow \mathbb{R}\) et \(a\in I\). On dit que \(f\) est continue à droite en \(a\) (respectivement à gauche en \(a\)) si et seulement si \(f(x) \xrightarrow[x \rightarrow a^+]{} f(a)\) (respectivement \(f(x) \xrightarrow[x \rightarrow a^-]{}f(a)\).
Soit \(a\in I\) un point intérieur (il existe \(\alpha > 0\) tel que \(]a-\alpha, a+\alpha[ \subset I\)). Une fonction \(f \in \mathscr F \left(I,\mathbb{R}\right)\) est continue en \(a\) si et seulement si elle est continue à droite et à gauche de \(a\).
La fonction \(\delta: \left\{ \begin{array}{ccl} \mathbb{R} & \longrightarrow & \mathbb{R} \\ x & \longmapsto & \begin{cases}0&\textrm{ si $x\neq 0$}\newline1&\textrm{ si $x=0$}\end{cases} \end{array} \right.\) n’est pas continue au point \(0\). On a bien \(\displaystyle{\lim_{x \rightarrow 0^-}\delta\left(x\right)} =\displaystyle{\lim_{x \rightarrow 0^+}\delta\left(x\right)} =0\) mais \(\delta\left(0\right)=1\).
(Prolongement par continuité).
Soit une fonction \(f\) définie sur \(I\) et un point adhérent \(a\in\overline I\) qui n’appartient pas à \(I\). On suppose que \(f(x) \xrightarrow[x \rightarrow a]{} l \in \mathbb{R}\). On définit alors la fonction \(\widetilde{f}\) sur \(\widetilde{I}=I\cup\left\{a\right\}\) par : \[\forall x\in \widetilde{I} \quad \bar{f}(x)= \begin{cases} f(x) & \textrm{ si $x\in I $}\newline l & \textrm{ si $x=a$} \end{cases}\] Cette fonction \(\widetilde{f}\) est continue au point \(a\) et est appelée prolongement de \(f\) par continuité au point \(a\).
Considérons la fonction \(f : \left\{ \begin{array}{ccl} \mathbb{R} [\star] & \longrightarrow & \mathbb{R} \\ x & \longmapsto & \sin x / x \end{array} \right.\). Puisque \(\sin x / x \xrightarrow[x \rightarrow 0]{} 1\), on peut la prolonger par continuité en une fonction définie sur \(\mathbb{R}\), \(\widetilde{f} : \left\{ \begin{array}{ccl} \mathbb{R} & \longrightarrow & \mathbb{R} \\ x & \longmapsto & \begin{cases} \sin x / x & \textrm{ si } x \neq 0 \newline 1 & \textrm{ si } x = 0 \end{cases} \end{array} \right.\).

Limites et relation d’ordre

(Passage à la limite dans les inégalités).
Soit une fonction \(f : I \mapsto \mathbb{R}\), un point \(a \in \overline{I}\) (éventuellement infini) et \(k \in \mathbb{R}\). On suppose que
  1. \(f(x) \xrightarrow[x \rightarrow a]{} l\).

  2. Il existe un voisinage \(V\) du point \(a\) tel que \(\forall x \in V \cap I\), \(k \leqslant f(x)\).

Alors \(k \leqslant l\).
Écrivons la démonstration dans le cas où \(a\) est \(l\) sont finis. Supposons par l’absurde que \(l < k\) et posons \(\varepsilon= k-l > 0\). Puisque \(f(x) \xrightarrow[x \rightarrow a]{} l\), il existe \(\eta_1 > 0\) tel que \(\forall x \in I\), \(\lvert x-a \rvert \leqslant\eta_1 \Rightarrow \lvert f(x)-l \rvert < \varepsilon\). Puisque \(V\) est un voisinage du point \(a\), il existe \(\eta_2 > 0\) tel que \(]a-\eta_2, a+\eta_2[ \subset V\). Posons alors \(\eta = \min(\eta_1, \eta_2)\). Puisque le point \(a\) est adhérent à \(I\), il existe \(x \in I\) tel que \(\lvert x-a \rvert \leqslant\eta\) et on doit avoir d’une part \(k \leqslant f(x)\) et \(\lvert f(x)-l \rvert < \varepsilon\) mais alors, \[k \leqslant f(x) < l + \varepsilon= l + (k-l) =k\] ce qui est absurde.
Le passage à la limite dans les inégalités ne conserve pas les inégalités strictes. Si sur un voisinage \(V\) de \(a\) on a \(k < f(x)\), on ne peut pas garantir que \(k < l\). Par exemple pour la fonction définie sur \(]0, 1]\) par \(f(x) = x\), \(\forall x \in ]0, 1]\), \(k=0 < f(x)\), \(f(x) \xrightarrow[x \rightarrow 0]{} 0=l\) et \(k=l=0\). On dispose bien sûr du théorème correspondant en remplaçant \(\leqslant\) par \(\geqslant\).
(Passage à la limite dans les inégalités).
Soient deux fonctions \(f,g : I \mapsto \mathbb{R}\), \(a \in \overline{I}\) et \(l_1,l_2 \in \mathbb{R}\). On suppose que
  1. \(f(x) \xrightarrow[x \rightarrow a]{} l_1\),

  2. \(g(x) \xrightarrow[x \rightarrow a]{} l_2\),

  3. il existe un voisinage \(V\) du point \(a\) tel que \(\forall x \in V \cap I\), \(f(x)~\leqslant~g(x)\).

Alors \(l_1 \leqslant l_2\).
Définissons la fonction \(h = g-f\). D’après les théorèmes généraux, \(h(x) \xrightarrow[x \rightarrow a]{} l_2-l_1\). D’après l’hypothèse \((3)\), sur un voisinage de \(a\), on a \(k=0 \leqslant h(x)\). D’après le théorème précédent, \(0 \leqslant l_2-l_1\) d’où \(l_1 \leqslant l_2\).
(Théorème des gendarmes).
Soient \(\alpha, ~f, ~ \beta\) trois fonctions définies sur un voisinage \(V\) d’un point adhérent \(a\in \overline{I}\) et \(l\in \mathbb{R}\). On suppose que
  1. \(\forall x \in V, \quad \alpha\left(x\right) \leqslant f(x) \leqslant\beta \left(x\right)\)

  2. \(\alpha\left(x\right) \xrightarrow[x\rightarrow a]{} l\) et \(\beta\left(x\right) \xrightarrow[x\rightarrow a]{} l\)

alors la fonction \(f\) admet une limite au point \(a\) et \(f\left(x\right) \xrightarrow[x\rightarrow a]{} l\).
Écrivons la preuve dans le cas où \(a\) est fini.
  1. Soit \(\varepsilon> 0\).

  2. Puisque \(\alpha(x) \xrightarrow[x \rightarrow a]{} l\), il existe \(\eta_1 > 0\) tel que \(\forall x \in I\), \(\lvert x-a \rvert \leqslant\eta_1 \Rightarrow \lvert \alpha(x)-l \rvert \leqslant\varepsilon\). De même, puisque \(\beta(x) \xrightarrow[x \rightarrow a]{} l\), il existe \(\eta_2 > 0\) tel que \(\forall x \in I\), \(\lvert x-a \rvert \leqslant\eta_2 \Rightarrow \lvert \beta(x)-l \rvert \leqslant\varepsilon\). Comme \(V\) est un voisinage du point \(a\), il existe \(\eta_3 > 0\) tel que \(]a-\eta_3, a+\eta_3[ \subset V\).

  3. Posons \(\eta = \min(\eta_1, \eta_2, \eta_3)\).

  4. Soit \(x \in I\) tel que \(\lvert x-a \rvert \leqslant\eta\). Puisque \(\lvert x-a \rvert \leqslant\eta \leqslant\eta_1\), \(l-\varepsilon\leqslant\alpha(x)\). Puisque \(\lvert x-a \rvert \leqslant\eta \leqslant\eta_2\), \(\beta(x) \leqslant l + \varepsilon\) et puisque \(\lvert x-a \rvert \leqslant\eta \leqslant\eta_3\), \(\alpha(x)\leqslant f(x) \leqslant\beta(x)\). On a finalement \(l-\varepsilon\leqslant\alpha(x) \leqslant f(x) \leqslant\beta(x) \leqslant l + \varepsilon\) d’où \(\lvert f(x)-l \rvert \leqslant\varepsilon\).

Le théorème des gendarmes se généralise aux limites infinies. Par exemple, si au voisinage de \(a\in \bar{I}\), on a
  1. \(f(x) \geqslant\alpha(x)\).

  2. \(\alpha\left(x\right) \xrightarrow[x\rightarrow a]{} +\infty\)

alors \(f\left(x\right) \xrightarrow[x\rightarrow a]{} +\infty\)
Attention, il ne faut pas confondre le théorème des gendarmes et le théorème de passage à la limite dans les inégalités. Le second permet d’affirmer l’existence d’une limite tandis que dans le premier l’existence de cette limite est présupposée.

Théorème de composition des limites

(Composition de limites).
Soient deux intervalles \(I \subset \mathbb{R}\), \(J \subset \mathbb{R}\) et deux fonctions \(f: I \rightarrow \mathbb{R}\) et \(g:J \rightarrow \mathbb{R}\) telles que \(f(I) \subset J\). Soient \(a\in \bar{I}\) et \(b\in\bar{J}\). On suppose que
  1. \(f\left(x\right) \xrightarrow[x\rightarrow a]{} b\)

  2. \(g\left(y\right) \xrightarrow[y\rightarrow b]{} l \in \overline{\mathbb{R}}\)

Alors \(\boxed{\left(g \circ f\right) (x) \xrightarrow[x\rightarrow a]{} l}\).
(1).
Écrivons la preuve dans le cas où \(a\) et \(l\) sont finis.
  1. Soit \(\varepsilon> 0\).

  2. Puisque \(g(y) \xrightarrow[x \rightarrow b]{} l\), il existe \(\alpha > 0\) tel que \(\forall y \in J\), \(\lvert y-b \rvert \leqslant\alpha \Rightarrow \lvert g(y)-l \rvert \leqslant\varepsilon\).

  3. Puisque \(f(x) \xrightarrow[x \rightarrow a]{} b\), il existe \(\eta > 0\) tel que \(\forall x \in I\), \(\lvert x-a \rvert \leqslant\eta \Rightarrow \lvert f(x)-b \rvert \leqslant\alpha\).

  4. Soit \(x \in I\) tel que \(\lvert x-a \rvert \leqslant\eta\). Comme \(y=f(x) \in J\) et que \(\lvert f(x)-b \rvert \leqslant\alpha\), on a \(\left|g\left(f(x)\right) -l \right| \leqslant\varepsilon\) d’où \(\lvert g\circ f(x) - l \rvert \leqslant\varepsilon\).

(Exponentiation \(\boxed{f^g}\)).
On déduit du théorème précédent les règles de passage à la limite dans une exponentiation \(f^g=e^{g\ln f}\). On suppose que \(f\) et \(g\) sont deux fonctions qui admettent comme limites respectives \(l\) et \(l'\).
 \(l'\) \(-\infty\) \(l'<0\) \(l'=0\) \(0<l'\) \(+\infty\)
\(l=0\) \(+\infty\) \(+\infty\) \(0\) \(0\)
\(0<l<1\) \(+\infty\) \(l^{l'}\) \(1\) \(l^{l'}\) \(0\)
\(l=1\) \(1\) \(1\) \(1\)
\(1<l\) \(0\) \(l^{l'}\) \(1\) \(l^{l'}\) \(+\infty\)
\(+\infty\) \(0\) \(0\) \(+\infty\) \(+\infty\)
  • Par exemple,

    • Lorsque \(x \rightarrow 0\), \(x^x=e^{x\ln x}\xrightarrow[x\rightarrow 0]{} 1\) car \(x\ln x\xrightarrow[x\rightarrow 0]{} 0\)

    • Lorsque \(x \rightarrow 0\), \(x^{{\scriptstyle 1\over\scriptstyle\ln x}}=e^{{{\scriptstyle\ln x\over\scriptstyle\ln x}}}=e\xrightarrow[x\rightarrow 0]{} e\).

  • Par exemple,

    • Lorsque \(x \rightarrow 0\), \(\left(1+x\right)^{{\scriptstyle 1\over\scriptstyle x}}=e^{{\scriptstyle\ln \left(1+x\right)\over\scriptstyle x}} \xrightarrow[x\rightarrow 0^+]{} e\) car

      \(\dfrac{\ln \p{1+x}}{x}\tendvers{x}{0^+}1\).

    • Lorsque \(x \rightarrow +\infty\), \(1^x=1\xrightarrow[x\rightarrow +\infty]{} 1\)

(Continuité de la composée de deux applications).

Soient deux intervalles \(I \subset \mathbb{R}\), \(J \subset \mathbb{R}\) et deux fonctions \(f: I \rightarrow J\) et \(g:J \rightarrow \mathbb{R}\) telles que \(f(I) \subset J\). On suppose que

  1. \(f\) est continue en \(a\).

  2. \(g\) est continue en \(b=f(a)\).

alors \(g\circ f\) est continue en \(a\).
C’est une conséquence immédiate du théorème précédent.

Image d’une suite par une fonction

(Théorème de la limite séquentielle).
On considère une fonction \(f: I \rightarrow \mathbb{R}\) et \(a\in \bar{I}\). Soit une suite \(\left(u_n\right)\) de points de \(I\) et \(l\in \bar{l}\). On suppose que
  1. \(u_n \xrightarrow[n\rightarrow +\infty]{} a\)

  2. \(f(x) \xrightarrow[x\rightarrow a]{} l\)

alors \(\boxed{f(u_n) \xrightarrow[n\rightarrow +\infty]{} l}\).
Supposons que \(l\) et \(a\) sont finis.
  1. Soit \(\varepsilon> 0\).

  2. Puisque \(f(x) \xrightarrow[x \rightarrow a]{} l\), il existe \(\eta > 0\) tel que \(\forall x \in I\), \(\lvert x-a \rvert \leqslant\eta_1 \Rightarrow \lvert f(x)-l \rvert \leqslant\varepsilon\).

  3. Puisque \(u_n \xrightarrow[n \rightarrow +\infty]{}a\), il existe un rang \(N \in \mathbb N\) tel que \(\forall n \in \mathbb N\), \(n \geqslant N \Rightarrow \lvert u_n-a \rvert \leqslant\eta\).

  4. Soit \(n \geqslant N\), puisque \(u_n \in I\) et \(\lvert u_n-a \rvert \leqslant\eta\), on a \(\left|f(u_n)-l\right| \leqslant\varepsilon\).

image

(Pour montrer qu’une fonction n’a pas de limite).
Soit \(f : I \mapsto \mathbb{R}\), \(a \in \overline{I}\) et \(l_1, l_2 \in \overline{\mathbb{R}}\). On suppose qu’il existe deux suites \((u_n)\) et \((v_n)\) de points de \(I\) vérifiant
  1. \(u_n \xrightarrow[n \rightarrow +\infty]{}a\), \(v_n \xrightarrow[n \rightarrow +\infty]{}a\)

  2. \(f(u_n) \xrightarrow[n \rightarrow +\infty]{}l_1\), \(f(v_n) \xrightarrow[n \rightarrow +\infty]{}l_2\)

  3. \(l_1 \neq l_2\)

alors \(f\) n’admet pas de limite au point \(a\).
Par l’absurde, s’il existait \(l \in \overline{\mathbb{R}}\) tel que \(f(x) \xrightarrow[x \rightarrow a]{} l\), puisque \(u_n \xrightarrow[n \rightarrow +\infty]{}a\), d’après le théorème de limite séquentielle, \(f(u_n) \xrightarrow[n \rightarrow +\infty]{}l\) et par unicité de la limite d’une suite, on aurait \(l = l_1\). De même, on aurait \(l = l_2\) et donc \(l_1=l_2\) ce qui est faux.
Considérons la fonction \(f : \left\{ \begin{array}{ccl} ]0, +\infty[ & \longrightarrow & \mathbb{R} \newline x & \longmapsto & \sin(1/x) \end{array} \right.\) et montrons qu’elle n’admet pas de limite en \(0\). Par l’absurde, supposons que \(f(x) \xrightarrow[x \rightarrow 0]{} l\). Introduisons les deux suites \((u_n) = (1/(n\pi))\) et \((v_n) = (1/(2n\pi + \pi/2)\). On calcule \(\forall n \in \mathbb N[\star]\), \(f(u_n) = 0 \xrightarrow[n \rightarrow +\infty]{}0\) et \(f(v_n) = 1 \xrightarrow[n \rightarrow +\infty]{}1\) et on aurait \(0 = 1\) ce qui est faux.
(Caractérisation séquentielle de la continuité en un point).
Soit une fonction \(f : I \mapsto \mathbb{R}\) et un point \(a \in I\). La fonction \(f\) est continue au point \(a\) si et seulement si pour toute suite \((x_n)\) de points de \(I\) convergeant vers \(a\), la suite \(\left(f(x_n)\right)\) converge vers \(f(a)\).
  • Soit \((x_n)\) une suite de points de \(I\) telle que \(x_n \xrightarrow[n \rightarrow +\infty]{}a\). Puisque \(f\) est continue au point \(a\), \(f(x) \xrightarrow[x \rightarrow a]{} f(a)\) et d’après le théorème de la limite séquentielle, \(f(x_n) \xrightarrow[n \rightarrow +\infty]{}f(a)\).

  • Cette démonstration peut être sautée en première lecture. Elle sera revue en deuxième année. Nous allons utiliser une technique classique en analyse : la construction d’une suite à partir d’une propriété de la forme \[\forall \eta > 0,~ \exists x\in I \dots\] En prenant \(\eta = 1/n\), on associe un élément \(x_n \in I\) et l’on construit ainsi une suite de points de \(I\). Pour obtenir une telle propriété, nous allons raisonner par l’absurde. Supposons donc que la fonction \(f\) n’est pas continue au point \(a\). La propriété \(f\) est continue au point \(a\) s’écrit à l’aide des quantificateurs : \[\forall \varepsilon> 0,~\exists \eta > 0,~ \forall x \in I,~ \lvert x-a \rvert \leqslant\eta \Rightarrow \lvert f(x)-f(a) \rvert \leqslant\varepsilon\] La traduction f n’est pas continue au point \(a\)   s’écrit en niant cette phrase : \[\exists \varepsilon> 0,~ \forall \eta > 0,~ \exists x \in I,~ \lvert x-a \rvert \leqslant\eta \textrm{ et } \lvert f(x)-f(a) \rvert > \varepsilon\] Il existe donc un réel \(\varepsilon> 0\) tel que \[\boxed{\forall \eta > 0,~ \exists x \in I,~ \lvert x-a \rvert \leqslant\eta \textrm{ et } \lvert f(x)-f(a) \rvert > \varepsilon}\] Pour tout entier \(n\) non nul, en prenant \(\eta = 1/n\), il existe un réel \(x_n \in I\) vérifiant \(\lvert x_n-a \rvert \leqslant 1/n\) et \(\lvert f(x_n)-f(a) \rvert > \varepsilon\). On construit ainsi une suite \((x_n)_{n \in \mathbb N[\star]}\) qui converge vers \(a\) puisque \(\lvert x_n-a \rvert \leqslant 1/n\). D’après (ii), notre suite image doit converger vers \(f(a)\) : \(f(x_n) \xrightarrow[n \rightarrow +\infty]{}f(a)\). Mais alors puisque \(\forall n \geqslant 1\), \(\varepsilon< \lvert f(x_n)-f(a) \rvert\), par passage à la limite dans l’inégalité on devrait avoir \(\varepsilon\leqslant\lvert f(a)-f(a) \rvert = 0\) ce qui est absurde.

Théorème de la limite monotone

(Théorème de la limite monotone (fonction croissante)).
Soient \((a,b)\in \overline{\mathbb{R}}^2\) et \(I = \left] a,b \right[\). Si une fonction \(f:\left] a,b \right[ \rightarrow \mathbb{R}\) est croissante, alors il y a deux possibilités.
  1. Si \(f\) est majorée, alors \(f\) admet une limite finie \(l\) lorsque \(x\) tend vers \(b\) et on a alors \(l= \sup_I f\).

  2. Si \(f\) n’est pas majorée, alors \(f(x) \xrightarrow[x\rightarrow b]{} +\infty\).

De même,

  1. Si \(f\) est minorée, alors \(f\) admet une limite finie \(l\) lorsque \(x\) tend vers \(a\) et \(l = \inf_I f\).

  2. Si \(f\) n’est pas minorée, alors \(f(x) \xrightarrow[x\rightarrow a]{} -\infty\).

(1).
Posons \(\mathscr E=\left\{f\left(x\right);~x\in \left]a,b\right[\right\}\). La partie \(\mathscr E \subset \mathbb{R}\) est non vide. Étudions les deux cas.
  1. Si la fonction \(f\) est majorée, alors la partie \(E\) est majorée et d’après l’axiome de la borne supérieure, elle possède une borne supérieure \(l \in \mathbb{R}\). Montrons qu’alors \(f(x) \xrightarrow[x \rightarrow b]{} l\).

    1. Soit \(\varepsilon> 0\).

    2. D’après le théorème de caractérisation de la borne supérieure ([caracterisation_borne_sup]), il existe \(y \in \mathscr{E}\) tel que \(l-\varepsilon\leqslant y \leqslant l\). Puisque \(y \in \mathscr{E}\), il existe \(x_0 \in \left]a,b\right[\) tel que \(y = f(x_0)\).

    3. Posons \(\eta = b-x_0 > 0\).

    4. Soit \(x \in I\) tel que \(\lvert x-b \rvert \leqslant\eta\), on a \(x_0 \leqslant x \leqslant b\). Puisque la fonction \(f\) est croissante, \(f(x_0) \leqslant f(x)\) et comme \(l\) est un majorant de \(\mathscr{E}\), on a également \(f(x) \leqslant l\). Finalement, \(l-\varepsilon\leqslant f(x_0) \leqslant f(x) \leqslant l\) d’où \(\lvert f(x)-l \rvert \leqslant\varepsilon\).

    image

  2. Si la fonction \(f\) n’est pas majorée, montrons que \(f(x) \xrightarrow[x \rightarrow b]{} +\infty\).

    1. Soit \(M > 0\).

    2. Puisque \(f\) n’est pas majorée, il existe \(x_0 \in \left]a,b\right[\) tel que \(M \leqslant f(x_0)\).

    3. Posons \(\eta = b-x_0 > 0\).

    4. Soit \(x \in I\) tel que \(\lvert x-b \rvert \leqslant\eta\).

    5. Puisque \(x_0 \leqslant x\) et que \(f\) est croissante, on a \(M \leqslant f(x_0) \leqslant f(x)\).

Multimédia: Comme pour les suites, l’utilisateur fixe son \(\varepsilon, B \dots\) et on obtient le \(\eta, A \dots\)

Si \(f\) est croissante et si \(f(x) \xrightarrow[x \rightarrow b]{} l \in \mathbb{R}\), alors \(\forall x \in ]a,b[\), \(f(x) \leqslant l\). En effet, d’après le théorème précédent, on est dans le premier cas et \(l\) est la borne supérieure de \(f\) donc un majorant de \(f\). Ce résultat est bien entendu faux si la fonction n’est pas croissante.

On a le théorème correspondant pour une fonction décroissante.

(Théorème de la limite monotone (fonction décroissante)).
Soient \((a,b)\in \overline{\mathbb{R}}^2\) et \(I = \left] a,b \right[\). Si une fonction \(f:\left] a,b \right[ \rightarrow \mathbb{R}\) est décroissante, alors il y a deux possibilités.
  1. Si \(f\) est majorée, alors \(f\) admet une limite finie \(l\) lorsque \(x\) tend vers \(a\) et on a alors \(l= \sup_I f\).

  2. Si \(f\) n’est pas majorée, alors \(f(x) \xrightarrow[x\rightarrow a]{} +\infty\).

De même,

  1. Si \(f\) est minorée, alors \(f\) admet une limite finie \(l\) lorsque \(x\) tend vers \(b\) et \(l = \inf_I f\).

  2. Si \(f\) n’est pas minorée, alors \(f(x) \xrightarrow[x\rightarrow b]{} -\infty\).

Le théorème de la limite monotone permet de justifier l’existence d’une limite sans la connaître explicitement. C’est un théorème d’existence abstrait très important en analyse.

Étude locale d’une fonction

Domination, prépondérance

Définitions

(Fonction dominée par une autre).
Soient \(f\) et \(g\) deux fonctions définies au voisinage de \(a\in\overline{\mathbb{R}}\). On dit que \(f\) est dominée par \(g\) au voisinage de \(a\) si et seulement si existe une fonction \(B\) définie au voisinage de \(a\) telle que
  1. \(f\left(x\right)=B\left(x\right)g\left(x\right)\) au voisinage de \(a\)

  2. \(B\) est bornée au voisinage de \(a\)

On note alors \(f\left(x\right)=\underset{x \rightarrow a}{O}\left(g\left(x\right)\right)\).
(Fonction négligeable devant une autre).

Soient \(f\) et \(g\) deux fonctions définies au voisinage de \(a\in\overline{\mathbb{R}}\). On dit que \(f\) est négligeable devant \(g\) au voisinage de \(a\) si et seulement si il existe une fonction \(\varepsilon\) définie au voisinage de \(a\) telle que

  1. \(f\left(x\right)=\varepsilon\left(x\right)g\left(x\right)\) au voisinage de \(a\)

  2. \(\varepsilon\left(x\right)\xrightarrow[x\rightarrow a]{}0\)

On note alors : \(f\left(x\right)=\underset{x \rightarrow a}{o}\left(g\left(x\right)\right)\)
\(f\) une fonction définie au voisinage de \(a\). Alors,
  • \(f\left(x\right)=\underset{x \rightarrow a}{O}\left(1\right) \quad\Longleftrightarrow\quad f\left(x\right)\textrm{ est bornée au voisinage de $a$}\)

  • \(f\left(x\right)=\underset{x \rightarrow a}{o}\left(1\right) \quad\Longleftrightarrow\quad f\left(x\right)\xrightarrow[x\rightarrow a]{}0\)

Propriétés

(Caractérisation pratique de \(f(x) = \grando{g(x)}\)).
Soit \(f\) et \(g\) deux fonctions définies sur un voisinage \(V\) de \(a\in\overline{\mathbb{R}}\). On suppose que \(g\) ne s’annule pas sur \(V\setminus\left\{a\right\}\). Alors \[\boxed{f\left(x\right)=\underset{x \rightarrow a}{O}\left(g\left(x\right)\right) \quad\Longleftrightarrow\quad \textrm{ la fonction } \dfrac{f}{g} \textrm{ est bornée au voisinage de $a$}}\]
  • Comme \(f\left(x\right)=\underset{x \rightarrow a}{O}\left(g\left(x\right)\right)\) il existe une fonction \(B\) bornée définie sur un voisinage \(V'\) de \(a\) (que l’on peut, quitte à travailler sur \(V\cap V'\) supposer égal à \(V\)) et vérifiant pour tout \(x\in V'\), \(f\left(x\right)=B\left(x\right)g\left(x\right)\). L’application \(f/g\) est donc définie sur \(V\setminus\left\{a\right\}\) et coincide avec \(B\) sur ce voisinage. Elle est donc bornée au voisinage de \(a\).

  • Réciproquement, si \(f/g\) est bornée au voisinage de \(a\), considérons un voisinage \(V\) de \(a\) sur lequel \(g\) ne s’annule pas sauf peut être en \(a\). Si \(x\in V\setminus\left\{a\right\}\), posons \(B\left(x\right)=\dfrac{f\left(x\right)}{g\left(x\right)}\) et posons \(B\left(a\right)=1\). La fonction \(\alpha\) est bien définie sur \(V\), et \(\forall x\in V\setminus\left\{a\right\},\quad f\left(x\right)=B\left(x\right)g\left(x\right)\). De plus \(B\) est bornée au voisinage de \(a\). Par conséquent \(f\left(x\right)=\underset{x \rightarrow a}{O}\left(g\left(x\right)\right)\).

(Caractérisation pratique de \(f(x) = \petito{g(x)}\)).
Soit \(f\) et \(g\) deux fonctions définies au voisinage sur un voisinage \(V\) de \(a\in\overline{\mathbb{R}}\). On suppose que la fonction \(g\) ne s’annule pas sur \(V\setminus\left\{a\right\}\). Alors \[\boxed{f\left(x\right)=\underset{x \rightarrow a}{o}\left(g\left(x\right)\right) \quad\Longleftrightarrow\quad \dfrac{f\left(x\right)}{g\left(x\right)}\xrightarrow[x\rightarrow a]{} 0}\]
  • Comme \(f\left(x\right)=\underset{x \rightarrow a}{o}\left(g\left(x\right)\right)\), il existe une fonction \(\varepsilon\) définie sur un voisinage \(V'\) de \(a\) (que l’on peut, quitte à travailler sur \(V\cap V'\), supposer égal à \(V\)) vérifiant, pour tout \(x\in V\), \(f\left(x\right)=\varepsilon\left(x\right)g\left(x\right)\) et \(\varepsilon\left(x\right)\xrightarrow[x\rightarrow a]{}0\). L’application \(f/g\) est définie sur \(V\setminus\left\{a\right\}\) et \(f(x)/g(x) = \varepsilon(x) \xrightarrow[x \rightarrow a]{} 0\).

  • Réciproquement, si \(\dfrac{f\left(x\right)}{g\left(x\right)}\xrightarrow[x\rightarrow a]{} 0\), posons \(\varepsilon\left(x\right)=\dfrac{f\left(x\right)}{g\left(x\right)}\) si \(x\in V\setminus\left\{a\right\}\) et posons \(\varepsilon\left(a\right)=0\). La fonction \(\varepsilon\) est bien définie sur \(V\), et on a  : \(\forall x\in V\setminus\left\{a\right\},\quad f\left(x\right)=\varepsilon\left(x\right)g\left(x\right)\). De plus \(\varepsilon(x) = f(x)/g(x) \xrightarrow[x \rightarrow a]{} 0\). Par conséquent \(f\left(x\right)=\underset{x \rightarrow a}{o}\left(g\left(x\right)\right)\).

Opérations sur les relations de comparaison

(Les relations \(o\) et \(O\) sont transitives).
Soient \(f\), \(g\) et \(h\) des fonctions définies au voisinage de \(a\in\overline{\mathbb{R}}\).
  • \(\left[f\left(x\right)=\underset{x \rightarrow a}{o}\left(g\left(x\right)\right) \quad \textrm{ et} \quad g\left(x\right)=\underset{x \rightarrow a}{o}\left(h\left(x\right)\right)\right]\Rightarrow f\left(x\right)=\underset{x \rightarrow a}{o}\left(h\left(x\right)\right)\)

  • \(\left[f\left(x\right)=\underset{x \rightarrow a}{O}\left(g\left(x\right)\right) \quad \textrm{ et} \quad g\left(x\right)=\underset{x \rightarrow a}{O}\left(h\left(x\right)\right)\right]\Rightarrow f\left(x\right)=\underset{x \rightarrow a}{O}\left(h\left(x\right)\right)\)

(Opérations sur les relations de comparaison).
\(\quad\) Soient \(f\), \(f_1\), \(f_2\), \(g\), \(g_1\) et \(g_2\) des fonctions définies au voisinage de \(a\in\overline{\mathbb{R}}\) :
    1. \(f_1 =\underset{x \rightarrow a}{o}\left(g\left(x\right)\right) \quad \textrm{ et} \quad f_2 =\underset{x \rightarrow a}{o}\left(g\left(x\right)\right) \Rightarrow f_1 + f_2 = \underset{x \rightarrow a}{o}\left(g\left(x\right)\right)\)

    2. \(f_1 =\underset{x \rightarrow a}{O}\left(g\left(x\right)\right) \quad \textrm{ et} \quad f_2 =\underset{x \rightarrow a}{O}\left(g\left(x\right)\right) \Rightarrow f_1 + f_2 = \underset{x \rightarrow a}{O}\left(g\left(x\right)\right)\)

    1. \(f_1 =\underset{x \rightarrow a}{o}\left(g_1\left(x\right)\right) \quad \textrm{ et} \quad f_2 =\underset{x \rightarrow a}{o}\left(g_2\left(x\right)\right) \Rightarrow f_1 f_2 = \underset{x \rightarrow a}{o}\left(g_1 g_2\left(x\right)\right)\)

    2. \(f_1 =\underset{x \rightarrow a}{O}\left(g_1\left(x\right)\right) \quad \textrm{ et} \quad f_2 =\underset{x \rightarrow a}{O}\left(g_2\left(x\right)\right) \Rightarrow f_1 f_2 = \underset{x \rightarrow a}{O}\left(g_1 g_2\left(x\right)\right)\)

Les preuves sont laissées en exercice. Vous pouvez supposer que les fonctions ne s’annulent pas sur un voisiange du point \(a\) pour utiliser les caractérisations précédentes.

Exemples fondamentaux

(Comparaison des fonctions usuelles).
Soient \(\alpha\), \(\beta\) et \(\gamma\) des réels strictement positifs.
  • En \(+\infty\) : \[\boxed{\left(\ln x \right)^\gamma = \underset{x \rightarrow +\infty}{o}\left(x^\alpha\right)} \quad \textrm{ et} \quad\boxed{x^\alpha = \underset{x \rightarrow +\infty}{o}\left(e^{\beta x}\right)}\]

  • En \(0\) et en \(-\infty\) : \[\boxed{\left|\ln x \right|^\gamma = \underset{x \rightarrow 0}{o}\left({\scriptstyle 1\over\scriptstyle x^\alpha}\right)} \quad \textrm{ et} \quad\boxed{e^{\beta x} = \underset{x \rightarrow -\infty}{o}\left({\scriptstyle 1\over\scriptstyle x^\alpha}\right)}\]

\(\,\) Aux bornes de l’intervalle de définition,

  • l’exponentielle l’emporte sur la puissance ,

  • la puissance l’emporte sur le logarithme.

Fonctions équivalentes

Définitions

(Fonctions équivalentes).
Soient \(f\) et \(g\) deux fonctions définies au voisinage de \(a\in\overline{\mathbb{R}}\). On dit que \(f\) est équivalente à \(g\) au voisinage de \(a\) si et seulement si \[\boxed{f\left(x\right)-g\left(x\right)=\underset{x \rightarrow a}{o}\left(g\left(x\right)\right)}\] On note alors \(f\left(x\right)\underset{x\rightarrow a}{\sim} g\left(x\right)\).
On a \(f\left(x\right)\underset{x\rightarrow a}{\sim} g\left(x\right)\) si et seulement s’il existe une fonction \(\varepsilon\) définie au voisinage de \(a\) telle que \(f(x) = \left(1+\varepsilon(x)\right) g(x)\) avec \(\varepsilon(x) \xrightarrow[x \rightarrow a]{} 0\).
(Caractérisation pratique de \(f(x) \underset{x \rightarrow a}{\sim} g(x)\)).
Soient \(f\) et \(g\) deux fonctions définies au voisinage de \(a\in\overline{\mathbb{R}}\). On suppose que \(g\) ne s’annule pas sur \(V\setminus\left\{a\right\}\). Alors \[\boxed{f\left(x\right)\underset{x\rightarrow a}{\sim} g\left(x\right) \quad\Longleftrightarrow\quad \dfrac{f\left(x\right)}{g\left(x\right)}\xrightarrow[x\rightarrow a]{} 1}\]
  • Comme \(f\left(x\right)\underset{x\rightarrow a}{\sim} g\left(x\right)\), il existe une fonction \(\alpha\) définie sur un voisinage \(V'\) de \(a\) (que l’on peut supposer égal à \(V\), quitte à considérer le voisinage \(V\cap V'\)) vérifiant, pour tout \(x\in V\), \(f\left(x\right)=\alpha\left(x\right)g\left(x\right)\) et telle que \(\alpha\left(x\right)\xrightarrow[x\rightarrow a]{}1\). L’application \(f/g\) est définie sur \(V\setminus\left\{a\right\}\) et \(f(x)/g(x) = \alpha(x) \xrightarrow[x \rightarrow a]{} 1\).

  • Si \(f(x)/g(x) \xrightarrow[x\rightarrow a]{} 1\), pour tout \(x\in V\setminus\left\{a\right\}\), posons \(\alpha\left(x\right)= f(x)/g(x)\) et \(\alpha\left(a\right)=1\). On définit ainsi une fonction \(\alpha\) sur le voisinage \(V\) avec \(\forall x\in V\setminus\left\{a\right\},~ f\left(x\right)=\alpha\left(x\right)g\left(x\right)\). De plus, \(\alpha(x) = f(x)/g(x) \xrightarrow[x \rightarrow a]{} 1\) donc \(f(x) \underset{x \rightarrow a}{\sim} g(x)\).

Propriétés

(Un équivalent donne localement le signe).
Si \(f\left(x\right)\underset{x\rightarrow a}{\sim} g\left(x\right)\) alors il existe un voisinage de \(a\) sur lequel \(f\) et \(g\) sont de même signe.
Comme \(f\left(x\right)\underset{x\rightarrow a}{\sim} g\left(x\right)\), il existe une fonction \(\alpha\) définie sur un voisinage \(V'\) de \(a\) vérifiant \(\forall x\in V',\quad f\left(x\right)=\alpha\left(x\right) g\left(x\right)\) avec \(\alpha\left(x\right)\xrightarrow[x\rightarrow a]{}1\). Puisque \(k=1/2 < 1\), d’après la transformation de limite en inégalité (théorème [thm:0718145314]), il existe un voisinage \(V'\) de \(a\) sur lequel \(\alpha(x) \geqslant 1/2 > 0\) et alors \(\forall x \in V\), \(f(x)\) est de même signe que \(g(x)\).
(Une fonction est équivalente à sa limite si celle-ci est non nulle et finie).
Soit \(f:I\rightarrow \mathbb{R}\) et \(a\in\overline I\). Alors \[\left[f\left(x\right)\xrightarrow[x\rightarrow a]{}l \quad \textrm{ et} \quad l\neq 0 \right]\Rightarrow f\left(x\right)\underset{x\rightarrow a}{\sim}l\]
Puisque \(f(x) \xrightarrow[x \rightarrow a]{} l\), d’après les théorèmes généraux, \(f(x)/l \xrightarrow[x \rightarrow a]{} 1\) ce qui signifie que la fonction \(f\) est équivalente à la fonction constante égale à \(l\) au voisinage du point \(a\).
(Deux fonctions équivalentes ont même limite).
Soit \(f,g:I\rightarrow \mathbb{R}\) et \(a\in\overline I\). Alors : \[\left[f\left(x\right)\underset{x\rightarrow a}{\sim}g\left(x\right) \quad \textrm{ et} \quad g\left(x\right)\xrightarrow[x\rightarrow a]{}l \right] \Rightarrow f\left(x\right)\xrightarrow[x\rightarrow a]{}l\]
Puisque \(f(x) \underset{x \rightarrow a}{\sim} g(x)\), il existe un voisinage \(V\) du point \(a\) et une fonction \(\alpha\) définie sur ce voisinage vérifiant \(\forall x \in V\), \(f(x) = \alpha(x)g(x)\) et \(\alpha(x) \xrightarrow[x \rightarrow a]{} 1\). D’après les théorèmes généraux sur les limites, \(f(x) = \alpha(x) g(x) \xrightarrow[x \rightarrow a]{} l\).
Attention, écrire \(f\left(x\right)\underset{x\rightarrow a}{\sim}0\) signifie que la fonction \(f\) est nulle sur un voisinage de \(a\), ce qui est possible, mais en général, lorsque vous écrivez \(0\) à droite d’un équivalent, vous commetez une erreur !
Soient \(a\in I\) et une fonction \(f\) définie sur \(I\). Si \(f\) est dérivable en \(a\) et si \(f'(a)\neq 0\), alors, au voisinage de \(a\), \[\boxed{f(x) - f(a) \underset{x\rightarrow a}{\sim} f'(a)(x-a)}\]
Comme \(f\) est dérivable en \(a\), son taux d’accroissement en \(a\) vérifie \(\dfrac{f(x)-f(a)}{x-a} \xrightarrow[x\rightarrow a]{} f'\left(a\right)\). Par conséquent, comme \(f'(a)\neq 0\), par opération sur les limites, on a \(\dfrac{f(x)-f(a)}{f'(a)(x-a)} \xrightarrow[x \rightarrow a]{} 1\) ce qui montre que \(f(x) - f(a) \underset{x\rightarrow a}{\sim} f'(a)(x-a)\).
Soient \(u\) une fonction définie au voisinage de \(a\in\overline{\mathbb{R}}\) et \(f\), \(g\) deux fonctions définies au voisinage de \(b\in\overline{\mathbb{R}}\). On suppose que
  1. \(u\left(t\right)\xrightarrow[t\rightarrow a]{}b\)

  2. \(f\left(x\right)\underset{x\rightarrow b}{\sim}g\left(x\right)\)

alors \(f\left(u\left(t\right)\right)\underset{t\rightarrow a}{\sim}g\left(u\left(t\right)\right)\).
Comme \(f\left(x\right)\underset{x\rightarrow b}{\sim} g\left(x\right)\), il existe une fonction \(\alpha\) définie sur un voisinage \(V'\) de \(b\) telle que \(\forall x\in V'\), \(f\left(x\right)=\alpha\left(x\right) g\left(x\right)\) et \(\alpha\left(x\right)\xrightarrow[x\rightarrow a]{} 1\). Soit \(V\) un voisinage de \(a\) tel que \(\forall t\in V,\quad u\left(t\right)\in V'\). On a alors \(\forall t\in V,\quad f\left(u\left(t\right)\right)=\alpha\left(u\left(t\right)\right) g\left(u\left(t\right)\right)\). De plus, comme \(u(t) \xrightarrow[t \rightarrow a]{} b\) et \(\alpha(x) \xrightarrow[x \rightarrow b]{} 1\), d’après le théorème de composition de limites ([thm:0719121126] page [thm:0719121126] ), \(\alpha\left(u(t)\right) \xrightarrow[t \rightarrow a]{} 1\) ce qui montre que \(f\left(u(t)\right) \underset{t \rightarrow a}{\sim} g\left(u(t)\right)\).
(Opérations sur les équivalents).
Soient \(f,g, \widetilde f, \widetilde g\) des fonctions définies au voisinage de \(a\in\overline{\mathbb{R}}\) telles que
  1. \(f\left(x\right)\underset{x\rightarrow a}{\sim} g\left(x\right)\)

  2. \(\widetilde f\left(x\right)\underset{x\rightarrow a}{\sim} \widetilde g\left(x\right)\).

Alors

  1. \(\boxed{ f(x)g(x) \underset{x \rightarrow a}{\sim} \widetilde f(x) \widetilde g(x) }\).

  2. Si la fonction \(\widetilde f\) ne s’annule pas sur un voisinage du point \(a\), il en est de même pour la fonction \(\widetilde g\) et alors \(\boxed{\dfrac{f\left(x\right)}{\widetilde f\left(x\right)} \underset{x\rightarrow a}{\sim} \dfrac{g\left(x\right)}{\widetilde g\left(x\right)}}\).

  3. Pour tout réel \(s\), si les fonctions \(f\) et \(g\) sont strictement positives au voisinage du point \(a\), \(\boxed{ \left[ f(x)\right]^{s} \underset{x \rightarrow a}{\sim} \left[ g(x)\right]^{s}}\).

Puisque \(f(x) \underset{x \rightarrow a}{\sim} g(x)\) et \(\widetilde f(x) \underset{x \rightarrow a}{\sim} \widetilde g(x)\), il existe un voisinage \(V\) du point \(a\) et deux fonctions \(\alpha\), \(\widetilde\alpha\) définies sur ce voisinage vérifiant \(f(x) = \alpha(x)g(x)\), \(\widetilde f(x) = \widetilde\alpha(x) \widetilde g(x)\) avec \(\alpha(x) \xrightarrow[x \rightarrow a]{} 1\) et \(\widetilde\alpha(x) \xrightarrow[x \rightarrow a]{} 1\). Nous pouvons alors écrire
  1. \(f(x)g(x) = \alpha(x)\widetilde\alpha(x) \widetilde f(x) \widetilde g(x)\) avec \(\alpha(x)\widetilde\alpha(x) \xrightarrow[x \rightarrow a]{} 1\) ce qui montre que \(f(x)\widetilde f(x) \underset{x \rightarrow a}{\sim} g(x)\widetilde g(x)\).

  2. Puisque \(\widetilde\alpha(x) \xrightarrow[x \rightarrow a]{} 1\), il existe un voisinage \(V'\) du point \(a\) sur lequel les fonctions \(\widetilde\alpha\) et \(\widetilde f\) ne s’annulent pas. Puisque \(\widetilde f(x)~=~\widetilde\alpha(x)~\widetilde g(x)\), la fonction \(\widetilde g\) ne s’annule pas sur \(V'\) et \(\forall x \in V'\), \(\dfrac{f(x)}{\widetilde f(x)} = \dfrac{\alpha(x)}{\widetilde\alpha(x)} \dfrac{g(x)}{\widetilde g(x)}\). D’après les théorèmes généraux, \(\alpha(x)/\widetilde\alpha(x) \xrightarrow[x \rightarrow a]{} 1\) ce qui prouve le résultat.

  3. On peut écrire sur un voisinge du point \(a\), \(\left[f(x)\right]^{s} = \left[\alpha(x)\right]^s \left[g(x)\right]^s\) et puisque \(y^s \xrightarrow[y \rightarrow 1]{} 1\), par composition de limites, \(\left[\alpha(x)\right]^s \xrightarrow[x \rightarrow a]{} 1\) ce qui prouve le résultat.

Il ne faut jamais
  1. Sommer des équivalents.

  2. Composer des équivalents. En particulier, il ne faut pas :

    1. Prendre des logarithmes d’équivalents.

    2. Prendre des exponentielles d’équivalents.

Par exemple \(x \underset{x\rightarrow +\infty}{\sim} x+1\) mais \(e^{x}\) et \(e^{x+1}\) ne sont pas équivalents quand \(x\) tends vers \(+\infty\) puisque \(\dfrac{e^x}{e^{x+1}}=e^{x-x-1}=e^{-1}\xrightarrow[x\rightarrow +\infty]{}e^{-1}\neq 1\).
(Équivalents classiques en \(0\)).
\[\boxed{\ln\left(1+x\right)\underset{x\rightarrow 0}{\sim} x }\qquad \boxed{e^x-1\underset{x\rightarrow 0}{\sim} x}\] \[\boxed{\sin x\underset{x\rightarrow 0}{\sim}x} \qquad \boxed{\tan x\underset{x\rightarrow 0}{\sim}x} \qquad \boxed{\mathop{\mathrm{sh}}{x}\underset{x\rightarrow 0}{\sim} x} \qquad \boxed{\tanh{x}\underset{x\rightarrow 0}{\sim} x}\] \[\boxed{\operatorname{arcsin} x\underset{x\rightarrow 0}{\sim}x} \qquad \boxed{\operatorname{arctan} x\underset{x\rightarrow 0}{\sim} x} \qquad \boxed{\mathop{\mathrm{argsh}}x\underset{x\rightarrow 0}{\sim}x} \qquad \boxed{\mathop{\mathrm{argth}}x\underset{x\rightarrow 0}{\sim} x}\] \[\boxed{\cos x-1\underset{x\rightarrow 0}{\sim}-{\scriptstyle x^2\over\scriptstyle 2}} \qquad \boxed{\mathop{\mathrm{ch}}x - 1\underset{x\rightarrow 0}{\sim} {\scriptstyle x^2\over\scriptstyle 2}} \qquad \boxed{\operatorname{arccos} x - {\scriptstyle\pi\over\scriptstyle 2}\underset{x\rightarrow 0}{\sim} -x }\] \[\boxed{\left(1+x\right)^\alpha-1\underset{x\rightarrow 0}{\sim} \alpha x} \quad \left(\alpha\in\mathbb{R}\right)\]
Les dix premières se démontrent en utilisant un taux d’accroissement, ou en utilisant la proposition [taux_accroissement_donne_equ]. Pour la onzième, \[\cos x-1 =\cos \left(2{\scriptstyle x\over\scriptstyle 2}\right)-1=-2\sin^2 {\scriptstyle x\over\scriptstyle 2}\underset{x\rightarrow 0}{\sim}-2 \dfrac{x^2}{4}=-\dfrac{x^2}{2}\] d’après l’équivalent usuel du sinus et par puissance d’équivalent. La douzième se prouve de même. Les deux dernières se prouvent encore en utilisant un taux d’accroissement. Pour l’avant dernière, on peut aussi utiliser la formule \(\forall x\in\left[-1,1\right],\quad \operatorname{arccos} x + \operatorname{arcsin} x= \pi/2\) et l’équivalent usuel d’arcsinus. La dernière peut encore se démontrer en passant en \(\exp-\ln\) et en utilisant les équivalents usuels pour \(\exp\) et \(\ln\)..
Attention, n’écrivez pas \(\cos(x) \underset{x \rightarrow 0}{\sim} 1-x^2/2\). Le résultat est vrai, mais on a plus simplement \(\cos(x) \underset{x \rightarrow x}{\sim} 1\) ! Il est conseillé de lire l’appendice pour comprendre ce qu’est un équivalent.

De manière plus générale,

Si \(f(x) \xrightarrow[x\rightarrow a]{}0\), au voisinage du point \(a\) \[\boxed{\ln\left(1+f\left(x\right)\right)\underset{x\rightarrow a}{\sim} f\left(x\right)} \qquad \boxed{\sin \left(f\left(x\right)\right)\underset{x\rightarrow a}{\sim} f\left(x\right)} \qquad \boxed{\tan \left(f\left(x\right)\right)\underset{x\rightarrow a}{\sim} f\left(x\right)}\] \[\boxed{\cos \left(f\left(x\right)\right)-1\underset{x\rightarrow a}{\sim}-{\scriptstyle\left(f\left(x\right)\right)^2\over\scriptstyle 2} } \qquad \boxed{e^{f\left(x\right)}-1\underset{x\rightarrow a}{\sim} f\left(x\right)} \qquad \boxed{\left(1+f\left(x\right)\right)^\alpha-1\underset{x\rightarrow a}{\sim} \alpha f\left(x\right)} \quad \left(\alpha\in\mathbb{R}\right)\]
Il suffit de combiner les résultats précédents et la proposition [compo_equ].

Une fois que vous avez assimilé les définitions de ce paragraphe, il est conseillé de lire l’appendice pour apprendre à utiliser en pratique les équivalents.

Propriétés globales des fonctions continues

Définitions et propriétés de base

Définitions

(Fonction continue sur un intervalle).
On dit qu’une fonction \(f\) est continue sur un intervalle \(I\) si et seulement si la fonction \(f\) est continue en chaque point de \(I\). Cette définition s’écrit avec les quantificateurs sous la forme suivante : \[\forall a\in I, \quad \forall \varepsilon>0 \quad \exists \eta>0 \quad \forall x\in I \quad \left|x-a\right|\leqslant\eta \Rightarrow \left|f\left(x\right)-f\left(a\right)\right|\leqslant\varepsilon\] On note \(\mathscr C\left(I\right)\) (ou \(\mathscr C^0\left(I\right)\), \(\mathscr C\left(I,\mathbb{R}\right)\), \(\mathscr C^0\left(I,\mathbb{R}\right)\)) l’ensemble des fonctions réelles continues sur l’intervalle \(I\).
  • La continuité en un point est une notion locale.

  • La continuité sur un intervalle est une notion globable.

  • Intuitivement, une fonction est continue sur un intervalle si et seulement si on peut tracer son graphe sans lever le crayon .

(Une fonction lipschitzienne est continue).
Si une fonction \(f~:~I\mapsto \mathbb{R}\) est lipschitzienne sur l’intervalle \(I\), alors \(f\) est continue sur l’intervalle \(I\).
Puisque \(f\) est lipschitzienne sur l’intervalle \(I\), il existe une constante \(k \geqslant 0\) telle que \(\forall (x,y)\in I^2\)\(\lvert f(x)-f(y) \rvert \leqslant k \lvert x-y \rvert\). Soit \(a \in I\). Montrons que la fonction \(f\) est continue au point \(a\).
  1. Soit \(\varepsilon> 0\).

  2. Posons \(\eta = \varepsilon/k > 0\).

  3. Soit \(x \in I\) tel que \(\lvert x-a \rvert \leqslant\eta\), \(\lvert f(x)-f(a) \rvert \leqslant k \lvert x-a \rvert \leqslant k\eta = \varepsilon\)

Opérations sur les fonctions continues

(Théorème d’opérations sur les fonctions continues).
  • Si \(f\) est continue sur \(I\) alors \(\left|f\right|\) est continue sur \(I\).

  • Une combinaison linéaire de fonctions continues sur \(I\) est continue sur \(I\).

  • La fonction produit de deux fonctions continues sur \(I\) est continue sur \(I\).

  • Si \(f\) et \(g\) sont continues sur \(I\) et si \(g\) ne s’annule pas sur \(I\) alors \({\scriptstyle f\over\scriptstyle g}\) est continue sur \(I\).

Les affirmations précédentes sont vraies en chaque point de \(I\) d’après les théorèmes généraux donc elles sont vraies sur \(I\).
\(\mathscr C\left(I\right)\) est un sous-espace vectoriel de \(\mathscr F(I,\mathbb{R})\).
(La composée de fonctions continues est continue).
Soient deux intervalles \(I\) et \(J\). Soit une application \(f\) continue sur \(I\) telle que \(f(I)\subset J\) et \(g\) une application continue sur \(J\). Alors la fonction \(g\circ f\) est continue sur \(I\).
la proposition est vraie en chaque point de \(I\) donc elle est vraie sur \(I\).

Les théorèmes fondamentaux

Le théorème des valeurs intermédiaires

(Théorème des valeurs intermédiaires (TVI)).
Soient \(I\) un intervalle de \(\mathbb{R}\) et une fonction \(f: I \rightarrow \mathbb{R}\). Soient deux points \((a,b)\in I^2\) tels que \(a<b\). On suppose que
  1. la fonction \(f\) est continue sur l’intervalle \(I\).

  2. \(f(a) \leqslant 0\) et \(f(b) \geqslant 0\).

Alors il existe un réel \(c\in[a,b]\) tel que \(\boxed{f(c)=0}\).
Puisque \(I\) est un intervalle et que \(a,b \in I\), on a \([a, b] \subset I\). Notons \(\mathcal{N}=\{x \in [a, b] \mid f(x) \leqslant 0 \}\). C’est une partie de \(\mathbb{R}\). Puisque \(a \in \mathcal{N}\), cette partie est non vide. De plus elle est majorée par \(b\) donc elle admet une borne supérieure \(c = \sup \mathcal{N}\). Montrons que \(f(c) = 0\).

  • D’après la caractérisation de la borne supérieure, \[\forall \varepsilon> 0,~ \exists x \in \mathcal{N},~ c-\varepsilon\leqslant x \leqslant c\] En prenant pour tout entier \(n\) non nul \(\varepsilon= 1/n\), il existe donc un réel \(x_n \in [c-1/n, c]\) vérifiant \(f(x_n) \leqslant 0\). On construit ainsi une suite de points de \([a,b]\) vérifiant \(x_n \xrightarrow[n \rightarrow +\infty]{}c\) et \(f(x_n) \leqslant 0\). Puisque la fonction \(f\) est continue au point \(c\), \(f(x_n) \xrightarrow[n \rightarrow +\infty]{}f(c)\) et par passage à la limite dans les inégalités, on a \(f(c) \leqslant 0\).

  • Puisque \(c\) est un majorant de \(E\), \(\forall x \in ]c, b]\), \(f(x) > 0\) et puisque \(f\) est continue à droite au point \(c\), \(f(x) \xrightarrow[x \rightarrow c^+]{} f(c)\). Par passage à la limite dans les inégalités, on en déduit que \(f(c) \geqslant 0\).

En conclusion, \(f(c) = 0\).
Le résultat est faux si la fonction est définie sur un ensemble \(A\) qui n’est pas un intervalle. Par exemple, la fonction \(f\) définie sur \([-2,-1] \cup [1, 2]\) par \(f(x) = -1\) si \(x \in [-2,-1]\) et \(f(x) = 1\) lorsque \(x \in [1, 2]\) est continue en tout point de \(A\), vérifie la deuxième hypothèse, puisque \(f(-2) < 0\) et \(f(2) > 0\) mais ne s’annule pas sur \(A\).
Plus généralement, on peut remplacer l’hypothèse \(H1\) par \(f(a)f(b) \leqslant 0\). Le théorème des valeurs intermédiaires est (comme le théorème de la limite monotone) un théorème qui permet de montrer l’existence d’objets de façon abstraite sans préciser leur valeur. On utilise pour cela une fonction auxiliaire bien choisie et on applique le TVI à cette fonction.
Soit \(f : [0, 1] \mapsto [0, 1]\) une fonction continue. Cette fonction admet au moins un point fixe \(x_0 \in [0, 1]\). Définissons la fonction auxiliaire \(g : \left\{ \begin{array}{ccl} [0,1] & \longrightarrow & [0,1] \newline x & \longmapsto & f(x)-x \end{array} \right.\). D’après les théorèmes généraux, la fonction \(g\) est continue sur le segment \([0,1]\). Puisque la fonction est à valeurs dans \([0, 1]\), \(f(0) \geqslant 0\) et \(f(1) \leqslant 1\) d’où \(g(0) \leqslant 0\) et \(g(1) \geqslant 0\). D’après le théorème des valeurs intermédiaires, il existe \(x_0 \in [0, 1]\) tel que \(g(x_0) = 0\), c’est-à-dire \(f(x_0) = x_0\).
Soit \(P : \mathbb{R} \mapsto \mathbb{R}\) une fonction polynomiale de degré impair. Vérifions qu’elle s’annule au moins une fois. En notant \(P(x) = a_{2n+1} x^{2n+1} + \dots + a_0\), avec \(a_{2n+1} \neq 0\), on obtient un équivalent simple de la fonction au voisinage de \(+\infty\) et de \(-\infty\), \(P(x) \underset{x \rightarrow \pm \infty}{\sim} a_{2n+1} x^{2n+1}\). Si \(a_{2n+1} > 0\), \(P(x) \xrightarrow[x \rightarrow -\infty]{} -\infty\) et \(P(x) \xrightarrow[x \rightarrow +\infty]{} +\infty\). La transformation de limite en inégalités donne l’existence d’un réel \(a < 0\) tel que \(P(a) \leqslant 0\) et d’un réel \(b > 0\) tel que \(P(b) \geqslant 0\). Puisque \(P\) est une fonction continue sur \([a, b]\), d’après le théorème des valeurs intermédiaires, il existe \(c \in [a, b]\) tel que \(P(c) = 0\).
(Recherche d’un zéro par dichotomie).
On considère une fonction continue \(f: [a, b] \mapsto \mathbb{R}\) telle que \(f(a) \leqslant 0\) et \(f(b) \geqslant 0\). On construit deux suites récurrentes \((a_n)\) et \((b_n)\) en posant \(a_0 = a\), \(b_0 = b\) et \[\forall n \in \mathbb N,\quad a_{n+1} = \begin{cases} a_n & \textrm{ si } f\bigl(\dfrac{a_n + b_n}{2}\bigr) \geqslant 0 \\ \dfrac{a_n + b_n}{2} & \textrm{ si } f\bigl(\dfrac{a_n + b_n}{2}\bigr) < 0 \end{cases} \quad b_{n+1} = \begin{cases} \dfrac{a_n + b_n}{2} & \textrm{ si } f\bigl(\dfrac{a_n + b_n}{2}\bigr) \geqslant 0 \newline b_n & \textrm{ si } f\bigl(\dfrac{a_n + b_n}{2}\bigr) < 0 \end{cases}\] Alors les deux suites \((a_n)\) et \((b_n)\) sont adjacentes et convergent vers une même limite \(c\) qui est un zéro de la fonction \(f\). Si l’on choisit de prendre \(a_n\) comme valeur approchée de \(c\), on obtient la majoration de l’erreur \[\forall n \in \mathbb N,~ \lvert c - a_n \rvert \leqslant\dfrac{b-a}{2^n}\]

On vérifie par récurrence que \(\forall n \in \mathbb N\), que \(a_n \leqslant b_n\) et que \((b_n-a_n) = (b-a)/2^n \xrightarrow[n \rightarrow +\infty]{}0\). Les deux suites \((a_n)\) et \((b_n)\) sont donc adjacentes et convergent vers la même limite \(c\in \mathbb{R}\). Puisque la fonction \(f\) est continue au point \(c\), d’après le théorème de la limite séquentielle, \(f(a_n) \xrightarrow[n \rightarrow +\infty]{}f(c)\) et \(f(b_n) \xrightarrow[n \rightarrow +\infty]{}f(c)\). On vérifie également par récurrence que \(\forall n \in \mathbb N\), \(f(a_n) \leqslant 0\) et \(f(b_n) \geqslant 0\). Par passage à la limite dans les inégalités, on obtient alors que \(f(c) \leqslant 0\) et \(f(c) \geqslant 0\) ce qui montre que \(f(c) = 0\). Puisque \(\forall n \in \mathbb N\), \(a_n \leqslant c \leqslant b_n\), \(\lvert c-a_n \rvert \leqslant(b_n-a_n) \leqslant(b-a)/2^n\) ce qui montre que \(a_n\) est une valeur approchée par défaut de \(c\) à \((b-a)/2^n\) près. De même, \(b_n\) est une valeur approchée par exces de \(c\) à \((b-a)/2^n\) près.
La preuve précédente fournit une autre démonstration plus constructive du théorème des valeurs intermédiaires. Elle fournit un algorithme simple et efficace qui permet de déterminer une valeur approchée d’un zéro de la fonction \(f\).
dicho := proc(f, a, b, eps) # f : une fonction définie sur le segment [a,b] # eps : une précision souhaitée # On suppose f continue sur [a,b] avec f(a) <= 0 et f(b) >=0 local A, B, C; A := a; B := b; C := (A + B)/2; while (B-A) > eps do # tant que la précision souhaitée n’est pas atteinte, calculer les termes suivants if f(C) < 0 then A := C; else B := C; fi; C := (A+B)/2; # après le n-ième passage dans cette boucle, A=a_n, B=b_n et C=(a_n+b_n)/2 od; # on sort de la boucle donc (B-A) <= eps C; #C est une valeur approchée de c à eps près end; f := x -> exp(x) - 2; dicho(f, -1., 2., 0.0001);
Multimédia: voir les segments de taille moitié qui encadrent le zéro au cours du temps

On dispose d’un énoncé équivalent du théorème des valeurs intermédiaires qui justifie son nom.

(Théorème des valeurs intermédiaires (deuxième forme)).
Soient \(a,b\in \mathbb{R}\) tels que \(a<b\). On suppose que :
  1. \(f\) est continue sur \([a,b]\).

alors \(f\left(x\right)\) prend toutes les valeurs intermédiaires entre \(f\left(a\right)\) et \(f\left(b\right)\) quand \(x\) parcourt \(\left[a,b\right]\). Autrement dit, si \(y_0\in \left[f(a),f(b)\right]\), alors il existe au moins un réel \(x_0\in [a,b]\) tel que \(\boxed{f(x_0)=y_0}\).
Supposons pour fixer les idées que \(f(a) \leqslant f(b)\), alors \(f(a) \leqslant y_0 \leqslant f(b)\). Définissons la fonction auxiliaire \[g~:~\left\{ \begin{array}{ccl} [a,b] & \longrightarrow & \mathbb{R} \newline x & \longmapsto & f(x)-y_0 \end{array} \right.\] Elle est continue sur \([a,b]\) d’après les théorèmes généraux et \(g(a) = f(a)-y_0 \leqslant 0\), \(g(b) = f(b)-y_0 \geqslant 0\). D’après le théorème des valeurs intermédiaires, il existe \(x_0 \in [a, b]\) tel que \(g(x_0) = 0\) et alors \(f(x_0) = y_0\).
Image d’un intervalle par une application continue L’image d’un intervalle par une application continue est un intervalle.
On suppose que \(f:I\rightarrow \mathbb{R}\) et que \(f\) est continue sur un intervalle \(I\). Soit \(J\) un intervalle de \(I\). Nous allons montrer que \(f\left(J\right)\) est encore un intervalle de \(\mathbb{R}\). Cela revient à prouver que pour tout \(y_1,y_2\in f\left(J\right)\), on a \(\left[y_1,y_2\right]\subset f\left(J\right)\). Soit \(y_1,y_2\in f\left(J\right)\). Il existe \(x_1\), \(x_2\in J\) tels que \(f\left(x_1\right)=y_1\) et \(f\left(x_2\right)=y_2\). Soit \(y\in\left[y_1,y_2\right]\). D’après le théorème des valeurs intermédiaires (deuxième forme), il existe \(x\in\left[x_1,x_2\right]\) tel que \(y=f\left(x\right)\). Par conséquent, \(y\in f\left(I\right)\). On prouve ainsi que \(\left[y_1,y_2\right]\subset f\left(J\right)\).

Fonction continue sur un segment

Le théorème suivant est fondamental en analyse.

(Théorème du maximum : une fonction continue sur un segment est bornée et atteint ses bornes).
Soit une fonction \(f~:~[a,b]\mapsto \mathbb{R}\) continue sur un segment. Alors la fonction \(f\) est bornée et atteint ses bornes \[\exists (c_1,c_2)\in [a,b]^2:\quad f(c_1)=\sup_{x\in [a,b]} f(x) \textrm{ et } f(c_2)=\inf_{x\in [a,b]} f(x)\]

(1).
La preuve utilise le théorème de Bolzano-Weierstrass. Il nous faut donc construire des suites. Pour cela nous allons utiliser la même technique que dans la démonstration du théorème .
  • Montrons que la fonction \(f\) est majorée : \[\exists M \in \mathbb{R} ,\quad \forall x \in [a, b],~ f(x) \leqslant M\] Nous raisonnons par l’absurde en supposant que la fonction n’est pas majorée : \[\forall M \in \mathbb{R} ,\quad \exists x \in [a, b],~ f(x) > M\] Soit un entier \(n \in \mathbb N\). En prenant \(M = n\), il existe \(x_n \in [a, b]\) vérifiant \(f(x_n) > n\). On construit ainsi une suite de points \((x_n)\) du segment \([a, b]\) telle que \(f(x_n) \xrightarrow[n \rightarrow +\infty]{}+\infty\). Puisque la suite \((x_n)\) est bornée, d’après le théorème de Bolzano-Weierstrass (), il existe une suite extraite \((x_{\varphi(n)})\) qui converge vers \(c \in \mathbb{R}\). Puisque \(\forall n \in \mathbb N\), \(a \leqslant x_n \leqslant b\), par passage à la limite dans les inégalités, \(a \leqslant c \leqslant b\). Mais la fonction \(f\) est continue au point \(c\) donc d’après la caractérisation séquentielle de la continuité, \(f(x_{\varphi(n)}) \xrightarrow[n \rightarrow +\infty]{}f(c)\). On obtient une contradiction puisque \(f(x_{\varphi(n)}) \xrightarrow[n \rightarrow +\infty]{}+\infty\).

  • Définissons la partie de \(\mathbb{R}\), \(\mathcal{F} = \{f(x);~ x \in [a, b]\}\). Elle est non vide puisque \(f(a) \in \mathcal{F}\). De plus, elle est majorée puisqu’on a vu que \(f\) était majorée. Elle admet donc une borne supérieure, \(M = \sup \mathcal{F} = \displaystyle{\sup_{I} f}\). Montrons que cette borne supérieure est atteinte. D’après la caractérisation de la borne supérieure, \[\forall \varepsilon> 0,~\exists x \in [a, b],~ \textrm{ tel que } \quad M-\varepsilon\leqslant f(x) \leqslant M\] Pour tout entier \(n\) non nul, en prenant \(\varepsilon= 1/n\), il existe \(x_n \in [a, b]\) tel que \[M-\dfrac{1}{n} \leqslant f(x_n) \leqslant M\] La suite \((x_n)\) étant bornée, d’après le théorème de Bolzano-Weierstrass, il existe une suite extraite \((x_{\varphi(n)})\) qui converge vers une limite \(c_1 \in [a, b]\). Puisque la fonction \(f\) est continue au point \(c_1\), \(f(x_{\varphi(n)}) \xrightarrow[n \rightarrow +\infty]{}f(c_1)\). On a d’autre part, \[\forall n \in \mathbb N[\star],\quad M-\dfrac{1}{n} \leqslant M-\dfrac{1}{\varphi(n)} \leqslant f(x_{\varphi(n)}) \leqslant M\] Par passage à la limite dans cette inégalité, on obtient que \(M \leqslant f(c_1) \leqslant M\) d’où \(M = f(c_1)\).

  • Pour montrer que \(f\) possède une borne inférieure et que cette borne inférieure est atteinte, on utilise les mêmes techniques. Vérifiez que vous avez bien compris la démonstration écrivant cette preuve.

En d’autres termes, une fonction continue sur un segment possède un maximum et un minimum : \[\displaystyle{\sup_{x\in I} f(x)} = \displaystyle{\max_{x\in I} f(x)} = f(c_1) \quad \displaystyle{\inf_{x\in I} f(x)} = \displaystyle{\min_{x\in I} f(x)} = f(c_2)\] On se sert souvent de ce théorème en analyse sous la forme suivante. Si \(f\) est une fonction continue sur un segment, la fonction \(\lvert f \rvert\) est également continue sur ce segment donc elle possède un maximum. On note \(\lVert f \rVert_{\infty} = \displaystyle{\sup_{x \in I} \lvert f(x) \rvert }\) ce maximum et il est atteint. Il existe \(c \in [a, b]\) tel que \(\lVert f \rVert_{\infty} = \lvert f(c) \rvert\).
(Image d’un segment par une application continue).
L’image d’un segment \([a,b]\) par une application continue est un segment et si \(m=\displaystyle{\inf_{[a,b]} f}\) et \(M=\displaystyle{\sup_{[a,b]} f}\) alors \(f\left([a,b]\right)=[m,M]\).
Puisque \(M\) est un majorant de \(f([a,b])\) et \(m\) un minorant de \(f([a,b])\), on a \(f([a,b]) \subset [m, M]\). Montrons que \([m, M] \subset f([a,b])\). Soit \(y \in [m, M]\). Comme les bornes sont atteintes, il existe \(c_1,c_2 \in [a, b]\) tel que \(M=f(c_1)\) et \(m = f(c_2)\). Un segment est un intervalle, donc d’après le théorème des valeurs intermédiaires (deuxième forme), puisque \(y \in [f(c_1), f(c_2)]\), il existe \(x \in [c_1, c_2] \subset [a, b]\) tel que \(y = f(x)\) ce qui montre que \(y \in f([a,b])\).

Fonctions uniformément continues

(Fonction uniformément continue).
Soit une fonction \(f~:~I \mapsto \mathbb{R}\) définie sur un intervalle \(I\). On dit qu’elle est uniformément continue sur \(I\) lorsque \[\forall \varepsilon> 0,\quad \exists \eta > 0:\quad ~\forall (x,y) \in I^2,\quad \lvert x - y \rvert \leqslant\eta \Rightarrow \lvert f(x) - f(y) \rvert \leqslant\varepsilon\] Le nombre \(\eta\) est indépendant des réels \((x, y)\) et s’appelle un module d’uniforme continuité.
(Lipschitz \(\Rightarrow\) uniformément continue \(\Rightarrow\) continue).
Soit \(f : I \mapsto \mathbb{R}\) une fonction définie sur un intervalle \(I\). \[f \textrm{ lipschitzienne sur} I\Rightarrow f \textrm{ uniformément continue sur } I \Rightarrow f \textrm{ continue sur } I\]
  1. Supposons \(f\) lispchitzienne sur \(I\), il existe \(k \geqslant 0\) tel que \(\forall (x,y) \in I^2\), \(\lvert f(x)-f(y) \rvert \leqslant k \lvert x-y \rvert\). Montrons que \(f\) est uniformément continue sur \(I\).

    1. Soit \(\varepsilon> 0\).

    2. Posons \(\eta = \varepsilon/ k > 0\).

    3. Soient \((x,y) \in I^2\) tels que \(\lvert x-y \rvert \leqslant\eta\), on a \[\lvert f(x)-f(y) \rvert \leqslant k\lvert x-y \rvert \leqslant k\eta = \varepsilon\]

  2. Supposons \(f\) uniformément continue sur \(I\) et montrons que \(f\) est continue sur \(I\). Soit \(a \in I\), montrons que la fonction \(f\) est continue au point \(a\).

    1. Soit \(\varepsilon> 0\),

    2. Puisque \(f\) est uniformément continue sur \(I\), il existe \(\eta > 0\) tel que \(\forall (x,y) \in I^2\), \(\lvert x-y \rvert \leqslant\eta \Rightarrow \lvert f(x)-f(y) \rvert \leqslant\varepsilon\).

    3. Soit \(x \in I\) tel que \(\lvert x-a \rvert \leqslant\eta\), on a bien \(\lvert f(x)-f(a) \rvert \leqslant\varepsilon\).

Le théorème suivant est un résultat important d’analyse.

(Théorème de Heine).
Une fonction continue sur un segment est uniformément continue sur ce segment.
Nous allons contstruire des suites et utiliser le théorème de Bolzano-Weirstrass. Nous devons montrer que \[\forall \varepsilon> 0,~\exists\eta >0,~\forall (x,y)\in [a,b]^2,\quad \lvert x-y \rvert \leqslant\eta \Rightarrow \lvert f(x)-f(y) \rvert \leqslant\varepsilon\] Raisonnons par l’absurde en supposant que cette propriété est fausse : \[\exists \varepsilon> 0,~\forall \eta > 0,~ \exists (x,y) \in [a,b]^2, \quad \lvert x-y \rvert \leqslant\eta \textrm{ et } \lvert f(x)-f(y) \rvert > \varepsilon\] Il existe donc un réel \(\varepsilon> 0\) tel que \[\forall \eta > 0,~ \exists (x,y) \in [a,b]^2,\quad \lvert x-y \rvert \leqslant\eta \textrm{ et } \lvert f(x)-f(y) \rvert > \varepsilon\] Soit \(n \in \mathbb N[\star]\), en prenant \(\eta = 1/n\), on peut trouver deux réels \((x_n, y_n) \in [a,b]^2\) vérifiant \(\lvert x_n-y_n \rvert \leqslant 1/n\) et \(\lvert f(x_n)-f(y_n) \rvert \geqslant\varepsilon\). On construit ainsi deux suites \((x_n)\) et \((y_n)\) de points du segment \([a, b]\). Puisque la suite \((x_n)\) est bornée, d’après le théorème de Bolzano-Weierstrass, on peut en extraire une suite convergente, \((x_{\varphi(n)})\) vers une limite \(c \in [a, b]\). Puisque \[\lvert y_{\varphi(n)}-c \rvert = \left|\left(y_{\varphi(n)}-x_{\varphi(n)}\right) + \left(x_{\varphi(n)} - c\right) \right|\leqslant \left| x_{\varphi(n)}-y_{\varphi(n)}\right| + \left|x_{\varphi(n)}-c\right| \leqslant \dfrac{1}{\varphi(n)} + \lvert x_{\varphi(n)}-c \rvert \leqslant\dfrac{1}{n} + \lvert x_{\varphi(n)}-c \rvert \xrightarrow[n \rightarrow +\infty]{}0\] la suite \((y_{\varphi(n)})\) converge également vers la même limite \(c\). Puisque la fonction \(f\) est continue au point \(c\), d’après la caractérisation séquentielle de la continuité, \(f(x_{\varphi(n)}) \xrightarrow[n \rightarrow +\infty]{}f(c)\) et \(f(y_{\varphi(n)})\xrightarrow[n \rightarrow +\infty]{}f(c)\). Mais comme \(\forall n \in \mathbb N\), \(\varepsilon< \left|f(x_{\varphi(n)}) - f(y_{\varphi(n)})\right|\), par passage à la limite dans les inégalités, on obtient que \(0 < \varepsilon\leqslant\left|f(c)-f(c)\right|=0\) ce qui est absurde.

Théorème de la bijection

(Théorème de la bijection).
Soit une application \(f:I \rightarrow \mathbb{R}\). On note \(J=f(I)\). On suppose que la fonction \(f\) est
  1. continue sur \(I\),

  2. strictement monotone sur \(I\).

Alors,

  1. \(J\) est un intervalle,

  2. \(f\) réalise une bijection de l’intervalle \(I\) vers l’intervalle \(J\),

  3. la bijection réciproque \(f^{-1} : J \mapsto I\) est une fonction continue sur \(I\), strictement monotone de même sens que \(f\).

D’après le théorème , on sait déjà que \(J\) est un intervalle. D’après le théorème [thm:strict_monotone] page [thm:strict_monotone], on sait déjà que \(f\) est bijective, strictement monotone de même sens que \(f\). Il nous reste à montrer que la bijection réciproque \(f^{-1}\) est continue sur \(J\). Soit \(X_0 \in J\), montrons que \(f^{-1}\) est continue au point \(X_0\). Notons \(x_0 = f^{-1}(X_0)\). Pour simplifier la preuve, nous supposerons que \(f\) est strictement croissante sur \(I\) et que \(x_0\) est un point intérieur à \(I\) (le cas où \(x_0\) est une borne de l’intervalle se traite de même).

  1. Soit \(\varepsilon> 0\).

  2. Puisque \(x_0\) est un point intérieur de \(I\), il existe un réel \(\varepsilon'\), \(0<\varepsilon' \leqslant \varepsilon\) tel que \([x_0-\varepsilon', x_0+\varepsilon'] \subset I\). Posons \(Y_1 = f(x_0-\varepsilon')\) et \(Y_2 = f(x_0+\varepsilon')\). Puisque \(f\) est strictement croissante sur \(I\), \(Y_1 < X_0 < Y_2\). Posons alors \(\eta = \min(X_0-Y_1, Y_2-X_0) > 0\).

  3. Soit \(X \in J\) tel que \(\lvert X-X_0 \rvert \leqslant\eta\), on a \(Y_1 \leqslant X \leqslant Y_2\) et puisque \(f^{-1}\) est croissante sur \(J\), \(f^{-1}(Y_1) \leqslant f^{-1}(X) \leqslant f^{-1}(Y_2)\), c’est-à-dire \(f^{-1}(X_0)-\varepsilon' \leqslant f^{-1}(X) \leqslant f^{-1}(X_0)+\varepsilon'\) ou encore \(\lvert f^{-1}(X)-f^{-1}(X_0) \rvert \leqslant\varepsilon'\leqslant\varepsilon\).

En résumé

Les parties [AnnexeC_Inegalites] page [AnnexeC_Inegalites] sur les techniques de majoration-minoration et la partie [AnnexeC_Equivalents] page [AnnexeC_Equivalents] sur les équivalents dans l’annexe [AnnexeC] sont, comme dans le chapitre précédent, toujours d’actualité.

Il faudra être capable de distinguer une propriété locale (vraie dans le voisinage d’un point) d’une propriété globale (vraie sur un intervalle).

Les énoncés suivants devront être parfaitement connus et quand on les utilisera, on vérifiera avec rigueur leurs hypothèses :

  1. Théorème des gendarmes.

  2. Théorème de la limite monotone.

  3. Caractérisation séquentielle de la continuité en un point.

  4. Théorème d’opérations sur les limites.

  5. Théorème d’opérations sur les fonctions continues.

  6. Équivalents usuels, relations de comparaison.

  7. Théorèmes des valeurs intermédiaires (sous ses différentes formes).

  8. Théorème du maximum.

  9. Théorème de Heine.

  10. Théorème de continuité de la bijection réciproque.

Bibliographie


    Barre utilisateur

    [ID: 66] [Date de publication: 10 décembre 2021 10:53] [Catégorie(s): Le cours de SUP ] [ Nombre commentaires: 0] [nombre d'éditeurs: 1 ] [Editeur(s): Emmanuel Vieillard-Baron ] [nombre d'auteurs: 3 ] [Auteur(s): Emmanuel Vieillard-Baron Alain Soyeur François Capaces ]




    Commentaires sur le cours

    Documents à télécharger

    Fonctions de la variable réelle
    Télécharger Télécharger avec les commentaires

    L'article complet